Monday, January 2, 2023

Koons on Aristotle and quantum mechanics

My review of Robert Koons’s excellent new book Is St. Thomas’s Aristotelian Philosophy of Nature Obsolete? appears at Public Discourse

204 comments:

  1. "What is most original in Koons’s book is his argument that quantum mechanics is best interpreted as vindicating the Aristotelian hylomorphist’s view of nature."
    “quantum hylomorphism,”
    No, the trend is always toward the bottom, as it were. Modern physics moves us ever more to reductionism. Particles themselves are merely illusions, rather, only fields are truly existent, particles merely being somewhat localized manifestations of fields.

    More speculatively, some theoretical physicists are working on strings as the fundamental reductionist building blocks.

    "The meat of the book is a sustained critique of several well-known interpretations of quantum mechanics"
    Right, clearly modern physics is highly incomplete. Poking a few holes in it is not difficult with a bit of research and merely doing so does not support hylomorphisim.

    "According to hylomorphism, the parts of a material thing are less fundamental to it than the whole"
    Indeed, hylomorphism holds that nonsensical position.
    "For example, hydrogen and oxygen exist in water, but only as constituents of the water rather than as entities in their own right. "
    You simply didn't reduce far enough, Dr. Feser. Consider the protons, electrons, and neutrons. Clearly they don't know they are part of hydrogen, oxygen, water, or anything else. They simply go wizzing about as they do, interacting as they do with whatever happens to be in the vicinity.

    Oh, yes, I know, you are saying I am now guilty of not reducing far enough, and right you are, because, of course, protons and neutrons are themselves composite, being composed of quarks. Yet even that does not reduce us to the bottom, say, fields of various sorts, which are manifested as electrons, and various sorts of quarks, among the the other residents of the particle zoo.

    Do these fields somehow have a "purpose", perhaps to make water wet, or to make air breathable, or to make a fusion reaction in the sun that ultimately warms my face mid-day? Such Aristotelian notions are at best quaint sepia colored legends.

    ReplyDelete
    Replies
    1. Welcome back StardustyPsyche. Your clear sightedness has been sorely missed.

      Delete
    2. Did you read the review? I haven't read the book, and I'm a complete layman on QM. But he states explicitly that on hylomorphism the lowest level of "prime matter" is indeterminate and formless (i.e. purposeless).

      "What such examples indicate is that matter at the smallest scales has precisely the kind of potentiality and indeterminacy that hylomorphism attributes to it, and that it is only the higher-level features of physical systems that actualize that potentiality and make of matter something determinate (which is the role hylomorphism attributes to form)." (Feser)

      I would suggest (re)reading what he said or clarifying why what you said here undermines his point.




      Delete
    3. I think you are missing Feser’s point in completely the opposite direction. He is obviously not arguing that the fault in the atomist’s logic is not reducing fat enough, but rather their entire notion that everything is reducible to the colorless, tasteless, heatless objects is off from the start. And he has provided a myriad of arguments for this in other locations.

      You are therefore missing Feser’s point again when you say that he is not reducing fat enough when he considers water more fundamental that it’s constituent elements. None of this entails a denial of science, chemistry, or physics by the way, and I think Feser would argue the opposite. The only way to make sense of the scientific process and discoveries of modern science is if our sense aren’t wrong. And the sense in which water is more fundamental has to do more with intentionality, and how these parts ultimately serve the purpose of the greater whole.

      Lastly, to your point that quantum mechanics has problems, that doesn’t entail that quantum hylomorphism is right. Well, you are correct, but I think again missing part of the point. There could be alternative correct interpretations, but this book is what is making the argument for quantum hylomorphism in detail. Feser is just reviewing the book in a few paragraphs. Also, I think the more important takeaway is that the problems with quantum mechanics are in its interpretation, as opposed to us missing the exact value of mass of some subatomic particle. The fact that we have no satisfactory interpretation for a scientific process we know with certainty to be at least partially mathematically correct (measurement problem) is good reason to consider other kinds of philosophical viewpoints. And the point Feser is making is that, unsurprisingly, the Aristotelian view of matter seems to fit what we observe in the quantum realm. I can’t speak to exactly how much I agree with this, as I am neither a PhD physicist or thomist philosopher, but I don’t think it’s a good counter argument to merely say, you just haven’t reduced far enough, the senses are mere illusions. Feser has argued against these views ad nauseam and they seem to kick the ladder out from underneath modern science

      Delete
    4. Stardusty, Are you back? I'm gonna. play " Ziiggy Stardust!"

      Delete
    5. Hi Stardusty, long time no see

      Delete
    6. Stardusty! I hope things have gone OK, or at least, dustily in your corner.

      Delete
    7. Thanks to AnonymousJanuary 2, 2023 at 4:59 PM, AnonymousJanuary 2, 2023 at 8:28 PM, ficino4mlJanuary 5, 2023 at 1:09 PM for the welcomes!

      "AnonymousJanuary 2, 2023 at 5:46 PM
      Did you read the review? "
      Yes, I read the review, which is where I got the quoted statements of Feser to which I responded.
      "I would suggest (re)reading what he said or clarifying why what you said here undermines his point."
      There is nothing in modern physics that would agree with the notion that assemblages of material somehow have purpose. Such assertions are just after the fact attempts to say "see, our ancient writers were correct all along". For example, Muslims do that with science as well, with a litany of supposed scientific predictions to be found in the Qur'an. Such claims, be they of Islam, Thomism, or other instances, quickly fall apart under closer examination.

      Modern physics is entirely reductionist, always seeking to account for large scale observations in terms of the aggregate of myriad interactions at the smallest scales.

      At no point in modern physics is there any notion of purpose somehow arising or emerging. The notion of associating purpose with large scale objects is purely human projection and has nothing to do with modern physics.

      Delete
    8. Oh, ooops, I just posted as Anonymous, well, thanks again to the folks who welcomed me back, the posting interface has changed slightly, it defaults to Anonymous and I neglected to change that..

      AnonymousJanuary 2, 2023 at 7:54 PM, I appreciate the long and thoughtful reply.

      "the point Feser is making is that, unsurprisingly, the Aristotelian view of matter seems to fit what we observe in the quantum realm."
      Indeed, that is a core error of Dr. Feser.

      Aristotle made a fundamental mistake in his view that all sublunary motion is in an impeding medium such that a moving object, absent a mover for it, will slow and stop, and it's motion will be lost.

      That notion, that fundamental Aristotelian error, is nowhere supported in modern physics.

      That core error of Aristotle led to the already scientifically falsified First Way by Aquinas. If that incorrect notion of Aristotle were correct then the First Way would actually be a very good argument for an unmoved mover. (here is one good source if you are interested in the 5 Ways, iteadthomam.blogspot.com/2011/01/first-way-in-syllogistic-form)

      Aristotle actually had the concept of Newtonian perpetual motion in his grasp, in Aristotle's book 4 of his physics, where Aristotle describes motion as continuing ad infinitum in the void.

      Bertrand Russel wrote some 100 years ago describing at length the errors of Aristotle, and described how tedious it was to read the ancient Aristotelian philosophers because nearly everything they said about physics was wrong.

      "I don’t think it’s a good counter argument to merely say, you just haven’t reduced far enough"
      Ok, you can think that, fine, but actually, yes, it is the case that in the example of hydrogen, oxygen, and water Feser projects notions that seem somehow vaguely reasonable but quickly break down when one considers further reductionist models. Hydrogen, oxygen, and water are all large scale structures relative to the underlying reality, which is just much smaller scale material whizzing about interreacting as they do with whatever happens to be in the vicinity, utterly oblivious to any larger scale structures and devoid of purpose or any potential to be a building block of purpose.

      Modern physics is devoid of purpose at any scale, so the assertion that modern physics in any way vindicates Aristotelian notions of purpose, or motion, or change...well, such assertions are just plain wrong.

      Delete
    9. Bertrand Russel wrote some 100 years ago describing at length the errors of Aristotle,

      Aristotle had a number of errors, of course. Russell's comments about him, however, are just about as foolish as the worst of Aristotle's errors: Aristotle had vastly many things right, and even Russell's comments about Ari's logic are basically stupid: Aristotle took a completely chaotic sea of predecessors who had nothing at all to offer on logic and - effectively by himself - created a systematic treatment. That the system he developed was LIMITED is true. The idea that his creating a limited system instead of the more refined system Russell came up with means Ari "held back" good logic is altogether idiotic in terms of the history of thought. Here is a cite that mentions a number of areas that Ari got right:
      https://talesoftimesforgotten.com/2020/10/22/aristotle-was-not-wrong-about-everything/

      Delete
    10. @Stardusty, anon here who asked if you read the article.

      What I should have said was,"did you *finish* the article?" Because at the very end of it was the quote from Feser that I provided, in which he argues for his position from the same fact you cited as contravening it. Namely, that the baseline level of reality is formless and indeterminate. If the baseline level of reality had a purpose (and therefore a form), it would *debunk* A/T, not confirm it! Your rhetorical question in your first comment ('Do these fields somehow have a "purpose"') suggests that you missed or forgot this fundamental point.

      In short, the claim that matter reduces to purposelessness is shared by both you and Feser. The real question is if/when form and purpose emerge again.

      Your second comment clarifies your stance that it never does. OK, to engage with that I'd probably need to read Koons' book and other books on QM interpretation and metaphysics and...it's beyond my pay grade. I just meant to address what I took to be a misunderstanding in your first comment based on how you concluded it.

      Delete
    11. Asock
      "What I should have said was,"did you *finish* the article?""
      Fair enough, I mean, just because one reads a part of the article does not mean one finished it, but yes, I finished it.

      If Feser holds that at base matter is without form and is indeterminate then that would violate the PSR, since an indeterminate process requires an effect without a reason. That is what intrinsic randomness would be, events happening for no reason, much less a sufficient reason.

      So, how does purpose emerge from purposelessness? Divine intervention? Aquinas argues for just that in his Fifth Way "Now whatever lacks intelligence cannot move towards an end, unless it be directed by some being endowed with knowledge and intelligence; as the arrow is shot to its mark by the archer. Therefore some intelligent being exists by whom all natural things are directed to their end; and this being we call God."

      There is nothing in modern physics that references a divine being to infuse purposeless fundamental material with purpose at some intermediate level, hence, modern physics does not align with Aristotelian physics in this regard.

      Now, if you wish, you can cherry pick a few areas of agreement, for example, in modern physics material moves, and Aristotle observed that material moves, so there you go.

      Here is one source for the physics of Aristotle. Aside from such trivial and superficial associations there is virtually no prediction of modern physics by Aristotle, and Aristotle got the core of physics wrong, which is why the argument for the first mover is unsound.
      archive.org/stream/aristotle-physics-book-III-and-IV-clarendon/aristotle-physics-book-III-and-IV-clarendon_djvu.txt
      IV. 8 TRANSLATION 214 b
      (4)
      214 b PHYSICS IV.8
      (4) ( a 19—22)

      Delete
    12. Not a fan
      Indeed, "Aristotle Was Not Wrong about Everything". Is that a reasonable basis to be an Aristotelian? Not wrong about everything?

      Even where his conclusions were wrong Aristotle made great progress by engaging in a process of observation, organization of data, and publishing his reasoning for his theories.

      Unfortunately, Aristotle was pretty much all wrong about physics. About all he got right were a few trivial observations. Modern physics has falsified Aristotelian physics, and therefore the First Way of Aquinas, which is based on Aristotelian physics.

      The Aristotelian arguments for a first mover depend on the notion that motion requires an external mover in the present moment. Modern physics has scientifically falsified that notion, thus the First Way has been scientifically falsified. No amount of cherry-picked trivial associations between Aristotle and modern observations can alter the simple fact that the argument for a first mover is based on a fundamental error of Aristotle.

      I am not familiar with your asserted unreasonable criticisms of Aristotelian logic by Bertrand Russel, fine, that is not what I was referring to with respect to the ways Russel pointed out the errors of Aristotle.

      Russel provided further explanation as to why the Aristotelian argument for a first mover is wrong, especially in his writings about mutual interactions. Aquinas pointed to the need for successive movers which cannot rationally go to infinity, which is a very good point. What Russel explained is that movers do not regress in a linear hierarchy, rather, movers interact mutually in a circular mutual process, thus terminating finitely the regression of movers Aquinas called for.

      In modern physics material is conserved, as expressed in the famous equation E=mc^2. Feser above calls for a first sustainer, yet modern physics and simple logic call for just the opposite. If X amount of material remains after time passes that is no change in the amount of existent material, therefore no first changer, no first sustainer, is called for.

      Feser has this clear and simple logic back to front.

      If material were to blink out of existence, then that would be a change calling for a changer. So, Feser seems to imagine an unseen changer continuously changing material back from spontaneously changing itself out of existence so that we observe no change.

      Further, Feser has, from time to time, described this notion of material simply blinking out of existence absent a sustainer. Thus, Feser advocates that Aquinas was wrong in stating in the First Way "It is therefore impossible that in the same respect and in the same way a thing should be both mover and moved, i.e. that it should move itself." Clearly, if a material thing were to blink itself out of existence it would "move" (change) itself from existing to not existing.

      Modern physics has scientifically falsified Aristotelian physics, thus, modern physics has falsified the Five Ways of Aquinas, which are based on Aristotelian (false) physics. Modern physics has scientifically falsified the A-T notions of a first sustainer, revealing such notions to be incoherent. Attempts by Koons and Feser to somehow find vindication of Aristotle in modern physics are confused, erroneous, and misguided at best.

      Delete
    13. Just a note or two. We sorta hope Ari was not wrong about everything. I have already stated my sense of his, and Thomas' status, vis-a-vis, physics. That much is well-settled. Insofar as he and TA were not physicists, a priori, their reputations as a. Thinker (Ari), and b. Ecclesiastic (Tom) remain unassailable, unless one wants to change the topic.Seems interdisciplinarianism has been around awhile. We just neglected to notice. Anyway, there is a time to every season, and, as with architects, thinkers come and thinkers go. They may or may not change our point(s) of view. And just so. Ken Wilber used to say that.

      Delete
    14. "Russel explained is that movers do not regress in a linear hierarchy, rather, movers interact mutually in a circular mutual process, thus terminating finitely the regression of movers Aquinas called for."

      Not that I doubt you, but can you cite where Russel argues this?

      Delete
    15. Anonymous January 18, 2023 at 10:47 AM,
      Yes, one particularly well known work by Russell is "On the notion of cause with applications to the free will problem" circa 1913, 1914. You can find a large number of publications and discussions of this piece.

      Russell did not bother to address Aristotle by name in that piece, he found reading and explaining the ancient philosophers of physics to be very tedious, as he found that scarcely a sentence they wrote was correct.

      His aim was to discuss modern physics as it was known at that time and to reject the common notion of causality.

      One example he used for his description of mutual interaction, as opposed to the ancient notion of hierarchical cause and effect, was gravity, possibly because that example is intuitively familiar to many.

      Here are a couple references:
      hist-analytic.com/Russellcause.pdf
      youtube.com/watch?v=7LpAqZIRc9Y

      Delete
    16. "At no point in modern physics is there any notion of purpose somehow arising or emerging. The notion of associating purpose with large scale objects is purely human projection and has nothing to do with modern physics."
      Wouldn't the fact that human action exists falsify the notion that purpose doesn't arise at any level? Also you seem to be a mereological nihilist, how do you square consciousness with your claim that only the fundamental simples exists and there's no objects with proper parts.

      Delete
    17. Anonymous January 23, 2023 at 9:39 AM,
      I think careful answers to your questions depend highly upon the precise definitions applied to terms you have employed.

      What does it mean to exist? No trivial question. Does running exist? Does running have mass, or any other static measurable property? Does running somehow exist Platonically? Or is it simply gibberish to even suggest running has an independent existence? I would say that material in the arrangement we identify as an animal exists and running is an identifiable process of such material. Material exists and form is a property of material that has no independent existence.

      Can we say for certain that such things as mereological simples do in fact exist? Certainly, modern physics is highly incomplete and such simples have yet to be identified and precisely described.

      Down thread Tom pointed out that in a molecule, say of water, what we call the constituent atoms are not precisely the same as they were as individual atoms, and indeed, forming of bonds is exothermic, so Tom is correct in that respect. Further, the orbitals of the electrons are of a different shape once the molecule is formed, so in that sense the water molecule is a different thing than simply the three free atoms were.

      So, do we know that a vaunted mereological simple retains its precise identity as such when joined with other asserted simples, or does the process of joining entail at least some small change in the constituents such that the former simples lose their precise identities as such in the process of joining, in a manner somewhat analogous to the changes hydrogen and oxygen undergo to form water?

      As for purpose, I did not precisely claim purpose in no sense ever arises at any level. That would seem to be a self defeating claim, similar to claiming that I do not exist in any sense or in any form.

      I said that purpose is a human projection, and I meant that with respect to inanimate objects, say, a rock. Is the purpose of the rock to provide a shelter for bugs, or to be filler in my gravel driveway, or to be a hammerstone to make a tool, or to be a weapon to bash your enemy with? The rock is just the rock, it has no intrinsic or ultimate or universal or absolute purpose. Any notion of purpose for the rock is a matter of human projection.

      Purpose is a human emotion, like the sense of ought, a personal feeling each of us may experience. Like running, purpose has no independent existence, rather, purpose is an identifiable process of material, the workings of the human brain.

      The Fifth way is arguably the worst in a list of arguments by Aquinas that start out bad at First and only get worse from there.

      Delete
  2. Physicist Stephen Barr in his First Things article ' Faith and. Quantum Physics" makes similar points as Koons.

    ReplyDelete
  3. There are some Christian apologists like Michael Jones from Inspiring Philosophy have argued that quantum mechanics proves idealism. What are your thoughts on this issue, Professor Feser?

    ReplyDelete
    Replies
    1. It might be that Michael Jones has just never heard of Aristotelian/Thomistic philosophy of nature and hylomorphism. I think he’s in going in the right direction, broadly speaking, by acknowledging that bottom-level physics are very mathematical and abstract in nature and he seems to suggest that nature is consisted of ‘ideas’ at the bottom level which seem to point towards a kind of divine conceptualism, but he uses terms that I don’t think would be acceptable to the hylomorphist.

      Delete
    2. Arriving at Idealism by doing philosophy is like arriving at the North Pole when trying to travel down south for the winter. You made a wrong turn somewhere. There is definitely logic to what he says but I think that an openness to idealism is a prerequisite, which is difficult if one has a proper metaphysical outlook going in.

      Delete
    3. As far as I know, Michael Jones has never mentioned A-T metaphysics on his channel. He puts out some really good content and he's a smart guy, but imo he's overly deferential to "modern consensus" in a lot of matters and has most likely written it off without consideration as "that outdated stuff that everyone knows was debunked by Francis Bacon or whoever" like most moderns.

      Regarding his idealism, I would say that's probably the second best interpretation of quantum mechanics out there.

      Let's say you've never really given A-T metaphysics much thought, and just generally take for granted the moderns' assumption that smaller things are more fundamental and that bigger things are aggregates of them rather than being substances with irreducible powers of their own.

      If you're a typical Reddit-tier intellectual lightweight atheist, you're liable to take the mere existence of increasingly small particles, and their lack of the properties that humans, animals, and objects of every day experience possess, as "proof" of eliminativism regarding those properties. See above in the comments to this very post for examples of that line of thinking.

      If you're a more thoughtful guy like Jones, you'll likely note that eliminativism is incoherent and invalidates all of our observations and all of our reasoning, and therefore cannot be true. Furthermore, you are likely to notice that our only knowledge of these increasingly tiny particles, and our only means of characterizing them, comes from utterly abstract mathematical equations that they seem to "obey." And since mathematical abstractions are mental objects, the purely mathematical and abstract natures of these particles, combined with the idea that the physical world consists of aggregates of them, is likely to lead you to the conclusion that the physical world is actually a bunch of mental abstractions in a Mind - aka idealism. It's strange, but unlike eliminativism it at least isn't totally incoherent and self-stultifying.

      Delete
  4. Aristotle couldn't even figure out that raw meat can't produce flies. His logic was that he let his wife leave some meat outside, didn't look at it for a few days, and concluded that it generated flies by itself. Why should we take him as an authority on logic when he couldn't even figure out something young children could?

    ReplyDelete
    Replies
    1. Try reading some Aristotle and come back to me

      Delete
    2. Ironically, if you think "Aristotle provided an erroneous hypothesis for observed phenomena, therefore we have no reason to take his metaphysics seriously," is a good argument, then surely you're the one that shouldn't be taken as "an authority on logic."

      Delete
    3. The idea that, because a person had incorrect beliefs in one field (biology), we can automatically disregard their work in an unrelated field (metaphysics) is absurd. That's like saying that, since Richard Dawkins is ignorant of basic philosophy of religion, we can ignore his biological work on evolution.

      Aristotle essentially founded the scientific study of biology. He got a lot right, more than anyone else up to his time had. Spontaneous generation was not a harebrained scheme aristotle came up with one afternoon. It was the science of his day (young children couldn't "figure it out" like you say, and they don't figure it out today. They are told why flies appear around dead meat by adults and believe it. They aren't running empirical experiments). The idea long predates him. And it survived after him. The theory wasn't decisively refuted until the time of Pasteur.

      Aristotle examined physics, metaphysics, biology, ethics, politics, psychology, logic, rhetoric, aesthetics, etc. It's a lame excuse to ignore his metaphysics because his biology isn't infallible.

      Delete
    4. @Anonymous, John

      Thinking that meat can spontaneously generate is not an error in biology, like when Darwin didn't understand genetics and believed in the protein transmission theory of inheritance. It is an example of sloppy thinking. Common sense tells you that "there's some meat, I looked away for several days, therefore the meat must have generated flies by itself!" is just sloppy, lazy thinking.

      And this isn't the only example of sloppy thinking. Aristotle later says that there are one, two, and then three dimensions of space, but no more. The idea of exploring multiple dimensions of space as mathematicians did is another example of extremely sloppy thinking.

      He was simply a sloppy thinker and a bad logician in general. Euclid was a better logician, but he too often made leaps of logic (assumed continuity) and presented dubious postulates (the parallel postulate was stated without any humble disclaimer like "this is just how my intuition thinks space works, it's not proven") There are lots of examples of his sloppiness, but these two are the most salient in my mind.

      Logic wasn't perfected until the mid-20th century. Prior to that, it was an extremely sloppy and half-baked subject.

      Delete
    5. Aristotle examined physics, metaphysics, biology, ethics, politics, psychology, logic, rhetoric, aesthetics, etc. It's a lame excuse to ignore his metaphysics because his biology isn't infallible.

      Another thing: Aristotle was an originalist. He taught that the best way to prevent a nation from going extinct was to create a written constitution and founding documents and stick as closely as possible to it. That's one more reason to know he's wrong.

      And a lot of Catholic metaphysics is just provably stupid nonsense that doesn't even originate from Aristotle. Like the idea that Star Trek style teleporters wouldn't cause a continuous transfer of consciousness but just kill you and create a replica. Everyone knows this is wrong.

      Delete
    6. "And a lot of Catholic metaphysics is just provably stupid nonsense that doesn't even originate from Aristotle. Like the idea that Star Trek style teleporters wouldn't cause a continuous transfer of consciousness but just kill you and create a replica. Everyone knows this is wrong."

      But...it would? Consciousness is deeply correlated with the activity and structure of the brain, neuroscience has made that pretty clear. If an area of the brain is damaged it affects consciousness. If you were to instantly disintegrate your brain, you would die because consciousness depends on the brain existing in that moment. It's sloppy thinking to say that 'everyone knows this is wrong,' especially since transferring consciousness is in the realm of science fiction, not fact.

      Delete
    7. @Infinite_Growth,

      How is it an example of sloppy thinking? You seem to be ignoring the fact that aristotle didn't come up with the theory, so it wasn't his own logic that led him to the idea. He was literally just recording observations he made and people told him about. He didn't reason to it by observing flies around meat. Have you every actually read his biological works?

      "Logic wasn't perfected until the 20th century".

      Even if that's true, once again it doesn't mean that aristotle's metaphysics were wrong. You also haven't pointed out anything wrong with aristotle's system of logic.

      "Another thing: Aristotle was an originalist. He taught that the best way to prevent a nation from going extinct was to create a written constitution and founding documents and stick as closely as possible to it. That's one more reason to know he's wrong."

      Have you ever actually read Aristotle? Because it sounds like you just read a summary of the politics in highschool. That's also an accurate summary of originalism, which is a theory if constitutional interpretation, not a political theory of state formation.

      Also, it's hilarious how you are going to call Aristotle's logic sloppy when you can't even construct a proper syllogism. "Aristotle was an originalist, therefore his metaphysics are wrong". Maybe you should start reading up on Aristotle's "sloppy logic". It seems like it would really help you.

      Delete
    8. He was simply a sloppy thinker and a bad logician in general. Euclid was a better logician, but he too often made leaps of logic (assumed continuity) and presented dubious postulates (the parallel postulate was stated without any humble disclaimer like "this is just how my intuition thinks space works, it's not proven")

      Ha ha, that's pretty funny!

      Wait, you were joking, weren't you? Euclid presents the parallel postulate as a POSTULATE and you're saying he should have said "it's not proven"? Do you even know what a postulate is? You don't? Hahahahaha... You just undermined your entire rant! Wow, self-destruction in just a few paragraphs.

      Delete
    9. *not* an accurate summary of originalism.

      Delete
    10. I'd also like to quip that it's absurd to believe that "logic wasn't perfected until the 20th century", when the 20th century's intellectual life was chock full of irrationalism (like the various political religions of the day) and nihilism/postmodernism. The most characteristic intellectual products of the 20th century are manifestly irrational and crazy.

      Delete
    11. @Cantus

      It is an objective fact that people were not able to create a 100% logical way of proving Euclidean geometry until the 20th century. Every formalization of Euclid prior to David Hilbert relied on subtle, unspoken axioms (e.g. Pasch's axiom is assumed and implicitly used in many of Euclid's proofs but cannot be proven from the five postulates) and intuition regarding continuity. So yes, it is provably true that an adult understanding of logic did not come until the 20th century.

      (like the various political religions of the day)

      Marxism was a theory of history and after Marx died nobody bothered trying to create a new theory of history. Francis Fukuyama being the only exception I know of.

      Delete
    12. Who is a troll? Example 1: Someone who argues that Aristotle didn't have a highly developed system of logic. To this very day the "standard" logic textbook will dive into some Aristotelian logic --- not to show how bad or out-of-date it is, but to show its force and practical use.

      Delete
    13. This comment has been removed by the author.

      Delete
    14. Infinite Growth

      "Like the idea that Star Trek style teleporters wouldn't cause a continuous transfer of consciousness but just kill you and create a replica. Everyone knows this is wrong."

      I am not sure this is wrong. Dr McCoy wasn't a big fan of the teleporter and i think if I lived in the Star trek universe, I wouldn't get into one.
      The problem is that your replica thinks it is you and there is no way to tell whether it really is you or not. Thomas Riker was actually a replica while Will Riker also continued to exist.
      or didn't he? Maybe Will Riker was also a replica.

      Delete
    15. It's very silly to say that "logic wasn't perfected until the 20th Century". For one thing, it's a very broad field. I assume you're talking about the Predicate Calculus (PC), but this system is far from perfect, and in many ways is inferior to the traditional logic even without taking into account modern extensions such as that invented by philosopher Fred Sommers, among others.

      The PC isn't user friendly because it's so far removed from natural language. Maybe that explains why it's hardly used except by a few computer scientists doing formal methods; it's certainly not used by mathematicians. Most of modern logic's "innovations" are very minor and consist mainly of "mathematization". Symbolization has its uses in terms of pattern finding, but it's been heavily overdone. Aristotle made the major conceptual breakthrough. There was simply no "formal" logic before him. In fact, logic is mostly studied as "subject" in its own right these days, usually an advanced topic in math classes, not as a tool to be used for seeking truth and evaluating arguments.

      It's usually asserted that the old logic is inferentially weaker than the PC, which is why the syllogism is usually relegated to short chapter (for historical interest) in the textbooks, if it's even mentioned at all. This is simply not the case - it's a massive con. Sure, the traditional syllogism IS limited, but the basic principle of syllogistic, the "Dictum de omni et nullo" (namely subsumption and substitution) can be applied to many different kinds of arguments, including those containing propositions having complex and relational terms. There is nothing that the PC can do that syllogistic can't. On the other hand, syllogistic can efficiently and easily evaluate arguments that the PC finds extremely cumbersome and unwieldy, such as numerical arguments.

      The context and agenda for the PC was the foundation of mathematics, which was in disarray at the time. It may be a good tool for that specialized application because its character is "ontologically explicit" insofar as it states clearly whether a given entity does or does not exist, but for everyday reasoning and philosophy, not so much.

      Delete
    16. I'd like to just appreciate the irony of Infinite Growth using an ad hominem attack to justify rejecting Aristotelian logic. In an article that talks about Aristotelian metaphysics, not his logic.

      IG, perhaps the one with the loose grasp of reality isn't Aristotle, but you.

      Delete
    17. I went to a Star Trek convention over the weekend dressed as a doctor, but the security guard didn't think I was the real McCoy.

      Delete
  5. Just finished this book tonight. I think it would be generous to say I understood half of it. All the same, I'm glad to see Thomists engaging at a detailed and technical level the relationship between philosophy of nature and current physical theory.

    One thing that was hit home for me is how pervasive microphysicalism is in our conceptualizing of physics and how this tendency bears some genuine responsibility for all the puzzles and seeming paradoxes quantum theory gives rise to.

    ReplyDelete
  6. "But as Koons points out, science itself is also undermined in either an atomist or a monist view. For there would also be nothing having the natures and purposes that we take our sense organs to have. Hence we would have no reason to believe that the world the senses present to us in any way corresponds to what is really out there—in which case, all the observational and experimental evidence on which science is based would be as illusory as everything else."

    That seems a non-sequitur to me and it is based on a misunderstanding of what "illusory" means in this context. A chair, e.g., would be illusory in the sense that at its basic level, it has no real "chairness". Nevertheless, the particular constitution of whatever fundamental particles make up the "chair" does exist and that constitution makes it the case that you can sit on a chair but it wouldn't be a good idea to sit on a razour blade even though the blade is made up of the same particles as the chair.
    The same goes for our sense organs. They are sense organs and not chairs because that's how their fundamental particles are organised.
    The "form" is not an illusion in the sense that it doesn't exist at all, it is an illusion because it doesn't exist as a somehow semi-independent quality.

    So, everyting is, in a sense, "really out there" and that includes our sense organs, which do have "natures and purpose" in the same way a chair has a nature and a purpose.

    Whether something has matter and form because it was shaped for a purpose or simply by coincidence does not make a difference. A stone can be as sharp as a razour blade even though it was not sharpened on purpose.

    ReplyDelete
    Replies
    1. The "form" is not an illusion in the sense that it doesn't exist at all, it is an illusion because it doesn't exist as a somehow semi-independent quality.

      The problem is that this really doesn't solve the problem; other than a chair, the problems of reducing the function of sensory organs to mere arrangements is more apparent. But that should be there minor quibble here. (It should be said though that the chair in itself has ascribed properties it mere parts lack, e.g. its shape, which makes the mereological nihilism you're hinting at quite implausible. Ever the more so for a biological organ)

      More importantly, the goal should be to undermine skepticism. And this is actually only possible if we suppose that a function is fulfilled here. An eyeball displaying illusions is only wrong (wrong as the normative judgment that it's not doing its job) insofar as there is an objectively correct mode of observation that the eyeball in question is incapable to represent. That is only something a form with a corresponding final cause is able to sustain. In either the atomist or the monist worldview, this idea has no room, since the arrangement is always only shape-wise, an imitation instead of a real member. If we take the two options seriously we have no reason to assume the observed data we're witnessing is actually reality representing.

      Of course we can't guarantee that with the Aristotelian conception either, after all they could malfunction right now. But the point is, similar to the debate about causation of contingent events, which worldview actually allows us rationally to employ phenomenal conservatism, and that's just what Koons describes here.

      Side note, purpose doesn't necessarily mean purposefully shaped or done that way, but also can refer to functions and usability. In that sense, the razor shaped stone can absolutely be directed towards one of its purposes (final causes) through using it as a razor.

      Delete
  7. Unfortunately, there is no Kindle version for this book, and I will have to wait it to be delivered with Amazon in Switzerland, hence my comments will be comments on (Feser’s) comment which is not the summit of the state of the art to discuss about this topic.
    I think it is rather ingenuous to wish to establish hylomorphism as the right paradigm to interpret quantum mechanic without doing the effort to analyze the logical structures that support the Aristotelian, the Newtonian, and the Quantum mechanical world.
    There has been a deep void in understanding the World from Illuminism to nowadays: (1) the deep misunderstanding of the nature of space and time on side, which became like a “box” where all bodies are contained, and which paradigm impedes very clever people to understand the meaning of physics at a reality layer; (2) the consequential shrinking of the Aristotelian/Thomistic logic into a poor subset, namely the Boolean logic, by avoiding to take into account the reality of the existence of things (medium term typically) to validate a reasoning; (3) the objective lack of understanding of what a form is, letting platonic idealism invade the field of scientific reflection (typical of the Descartes pretention to find the soul, which is the form of the human body, inside the pineal gland).
    Hence there is no wonder that people do not actually understand that general and local theories of relativity are not the physics of a space-time universal box (i.e., how could the universe be a big block of space and time, while neutrinos and photons, in their own reference’s frame exist for zero second in an infinite space, while for each of us all what we experience and even think is in our physical past, etc. etc.); the symmetries that we find (which are actually the expression of the law of physics) are never perfectly realized in the nature (e.g., time cannot be reversed); or when reflecting on quantum mechanics the paradigmatic incapacity to understand that the form of a system, included its potentiality, is not an element of space and time and that it is impossible to decorrelate a potentiality from the objective possible ends of the considered quantum system.
    The Newtonian world is an impoverished world which thinks essentially in terms of BArbArA, while science proceeds falsifying reasoning like BOcArdO, and while everyday people actually uses essentially a DArII logical structure, and Quantum Mechanic the similar DArAptI.
    What defines univocally a form is the final cause of it, which is typical of the Quantum Approach of measurement: in contrast, reductionism, which is fine for engineering purposes, cannot pretend to define, per construction, the final cause, even though it allows to understand “the how” this last one is made "reachable", it cannot trigger per se that one.
    The final cause is, at the end, what gives form to the form included the Quantum form within a world which is driven by Aristotelian logic and, all of that, without paradoxes.

    ReplyDelete
  8. This is speculative, at best, IMHO. Working backwards, there are no paradoxes---only discrepancies we are unprepared to work out. Quantum mechanics contains some, and, as we see it, we cannot resolve them. There are matters of physics, and, matters of metaphysics. Aristotle and Thomas Aquinas were not physicists, so far as I know. Neither were they, necessarily, metaphysicists---if that would have mattered. It puzzles me when writers try to correlate very old knowledge and application with newer information and experience. When did science an mathematics begin?

    My mind goes to Copernicus and Galileo, followed by DesCartes and Newton. Clearly, these were not the only ones. In whatever case, I wish Mr. Koons well with his book. Everyone has their own album to do.

    ReplyDelete
    Replies
    1. Paul,

      Aristotle and St. Thomas not metaphysicists? That would be news to Aristotle as he wrote a work entitled "metaphysics" and news to St. Thomas who wrote a commentary on this work. It would also be news to one of the most prominent Thomist scholars of the past generation, Fr. John Wippel who wrote the book The Metaphysical Thought of Thomas Aquinas. Perhaps you are puzzled because you are badly misinformed.

      Regarding Physics, this word was used in a different sense among classical and medieval realists than it is used in contemporary science, but Aristotle wrote a work entitled Physics and St. Thomas also wrote a commentary on this as well. Oops again, eh.

      Regarding the origins of modern science, you might read the two volume work of William Wallace who taught at CUA and had advanced degrees in both physics and theology. As he exhaustively demonstrates, the origins of modern science have their roots not in Galileo and Kepler and Copernicus, but rather in late medieval scholastic thinkers like Roger Bacon, Albertus Magnus, and Robert Grosseteste (among others). The Oxford historian of science AC Crombie also recognized these origins in his two volume work on the ancient and medieval origins of modern science. While there were significant developments with later figures, the suggestion that both science and mathematics began with modern thinker is, once again, badly misinformed. Your mind goes to Galileo and Copernicus because you are unfamiliar with the work of some of the most prominent historians of science writing in the past 75 years.

      Delete
    2. Carrying forward this comment:

      Your mind goes to Galileo and Copernicus because you are unfamiliar ...

      with the actual development of thought in history.

      To take just ONE aspect: Descartes' algebraic techniques were preceded by al-Khwārizmī and Viete, and before them enormously important strides were made by the ancients such as Apollonius, Archimedes, and Diaphantus, even though not specifically in algebraic form. Galileo's advances were not in principal mathematical, so he cannot be put into that line of development. To rest "mathematics" on Descartes is to be completely wrong on how algebraists moved the ball forward from the ancients through medievals to the Renaissance rather than Descartes creating the foundational ground-work of "real" math. Descartes' 17th century work simply could not have been made without the prior foundations laid with 2000 years of math.

      Delete
  9. Hi Ed,

    In your review, you write: "According to hylomorphism, the parts of a material thing are less fundamental to it than the whole—and, indeed, exist in the whole only as potentially independent things rather than as actual entities in their own right."

    However, in "Five Proofs of the Existence of God" (San Francisco: Ignatius Press, 2017), you write: "And a composite is less fundamental than its parts in the sense that its existence presupposes that its parts exist and are put together in the right way" (p. 70).

    You can't have it both ways. If the parts of a thing are merely potential, relative to the whole, then they cannot be ACTUALLY put together in the right way, as they would have to be in order to compose a whole.

    You also write in your review that "the larger universe is merely the sum of the ordinary middle-range objects that make it up—contrary to monism’s claim that the universe as one big lump is the only fundamental entity (with ordinary objects being mere modifications of it)." OK, so what are the ordinary middle-range objects that make up the universe? Is a galaxy an object? Is a star an object? What about a planet, an asteroid, a boulder, and a clump of earth? Do they or don't they have a substantial form of their own? If hylomorphism is true, then the answer cannot be "yes and no." It has to be one or the other, because a substantial form cannot be half-actual.

    The metaphysics of common sense may tell us that particles are real and fundamental, but a physicist will tell you that particles are best viewed as excitations of fields. See here: https://physics.stackexchange.com/questions/122570/what-is-more-fundamental-fields-or-particles

    Contrary to what you claim, quantum mechanics offers no support for the Aristotelian hylomorphist’s view of nature. Aristotle did not view a thing's position and momentum as observer-dependent properties of that thing, which are merely potential until it interacts with a middle-sized object. For Aristotle, a thing's position is absolute, not relative. See this article: https://physics.bgsu.edu/p433/Spacetime5.html

    I should point out that while it's easy for Koons to criticize this or that interpretation of quantum mechanics, there are about 15 mainstream interpretations, not to mention the minority interpretations. See the following table: https://en.wikipedia.org/wiki/Interpretations_of_quantum_mechanics#Comparisons

    How would Koons answer the questions in the table?

    Finally, Catholic doctrine does not require us to suppose that humans and other living things are fundamental. What it says is that they are real, and that their reality cannot be reduced to that of their constituents. We don't need to adopt hylomorphism; we just need to reject reductionism. Cheers.

    ReplyDelete
    Replies
    1. Vincent, at the risk of being a scold, you should really know better than to throw out some of these criticisms, most of which are either misunderstandings or irrelevancies. You've been around here long enough to be able to get the picture:

      1) Parts stand in potential to the whole qua substance. The whole stands in potential to the parts qua composite. The claim is not that the whole is prior to the part in literally every respect. So of course the Thomist can have both statements, because the statements aren't claiming incompatible things.

      2) Who knows whether nebulae, galaxies, etc. are substances? My suspicion is no, but that the question is a difficult one to answer as a matter of fact shouldn't much be surprising. Nothing about hylomorphism says it must be easy to discern what is a substance and what is not.

      3) Koons explicitly rejects the idea that particles are fundamental, so this is a moot point.

      4) For heaven's sake, not this objection of "Aristotle's science held different conclusions and paradigms from contemporary science" again. We know. But this is not what Koons is setting out to defend. You assert that QM does nothing to support a hylomorphic philosophy of nature and then go on to speak about, not matter and form, but certain niceties of Aristotle's account of accidents (i.e. place, relation, position/posture, etc.) which don't directly bear on hylomorphism at all.

      Delete
    2. However, in "Five Proofs of the Existence of God" (San Francisco: Ignatius Press, 2017), you write: "And a composite is less fundamental than its parts in the sense that its existence presupposes that its parts exist and are put together in the right way" (p. 70).

      He says there is a sense in which the parts are more fundamental. On the very page before that he says "There is a sense in which, in each of these cases, the parts are less fundamental than the whole" and then goes on to give examples of why.

      I swear, it seems like you're addicted to being mindlessly petty and contrarian on Ed's blog over every little thing he says, presumably because you're still mad at him over his criticisms of common ID approaches a few years back, even when the most cursory reading of his remarks makes it obvious that you're butchering his point, and even when your nitpicks undermine your *own* theistic metaphysics where they overlap with his.

      Delete
    3. Also:

      If the parts of a thing are merely potential, relative to the whole, then they cannot be ACTUALLY put together in the right way, as they would have to be in order to compose a whole.

      Ed didn't say that the parts are potential "relative to the whole." He said they are only potentially *independent* of the whole. It's right there in your first quote.

      In other words, what makes an eye an eye is the role it plays in an organism (namely seeing), which it couldn't do if it weren't part of the whole. Yet at the same time, an eye is a *potentially* independent thing, inasmuch as it could be plucked out of an organism, in which case it would no longer be a part of the organism but an actually independent bit of matter without a biological function in its new context (and thus, technically, no longer an eye).

      Again, this is just really basic and obvious stuff that you're going out of your way to misread. You've become like the mirror image of the no-concessions policy of Darwinists when talking to Ed.

      Delete
    4. When related to quantum mechanics it makes no sense to speak decently of the parts and the whole while doing the economy to really understand what the difference between pure and mixed states is: which, actually, is what leads to the deep understanding of the Bell’s inequalities.
      It is also meaningless to talk about Aristotelian forms if we tend to interpret them like “shapes” having measurable extensions and movements: potency is, per definition, not actual and cannot be described in terms of measurements. But potency is real, like QM shows, and the interaction of a quantum system with an instrument can be perfectly described in term of measurements.
      To understand that a potency is not localized in any point of space and time till it interacts with an instrument, we can give the ideal example of a bridge long 300’000 km, one quality of the bridge is to be breakable, the measurement of this property will consist in trying to break that bridge, if the measurement is successful that bridge will be broken in whatever point, then we will not have a breakable bridge but a broken bridge, i.e. the actual state of the bridge will bridge will change from breakable into broken and this concern the whole bridge, independently from it length and independently from the time needed for the information to be shared at both ends of the bridge: whatever will be measured at that time is simply the fact that that bridge is broken.
      There is a paradigm that twists the mind of too many people, even physicists, when thinking about a physical system: in the Aristotelian world as well as in the Quantum world which just need two assumption to perfectly define a physical system, namely the fact that (a) for each physical state it exists another which is orthogonal to itself and (b) that movement consists in loosing some properties (which become orthogonal) while getting other which were potentially accessible; in the Newtonian/classical physics there is a third assumption which is claimed, but unnecessary in Quantum and Aristotelian physics, which is the fact that one requires (c) that the result of an experience must be either actual or potential.
      The fact that there is no need of (c) in the “daily’s everybody’s physics” like the Aristotelian one as well as in the Quantum one, shows that there is unity of being and logic and little solution of continuity between both interpretative bodies.

      Delete
  10. Lots of lively participation---and contentiousness---on this blog. Were I Dr. Feser, I would be pleased. I have considered the critique of my last comment. I mentioned Aristotle and Thomas Aquinas, claiming they were not physicists. I'll stand by that, unless philosophers and ecclesiastics are also physicists. I don't think so. On the metaphysical plane, spelling intentional, I suppose religion takes a seat. Insofar as metaphysics and religion both deal with matters that are not strictly evidentiary. Are religion and philosophy neighbors on the plane? It would seem so. They certainly are in university humanities departments, had you not noticed. And so, is religion philosophy? There is evidence of this. Belief, after Davidson, is a propositional attitude, and both religion and philosophy are propositional. Carry on.

    ReplyDelete
    Replies
    1. Insofar as metaphysics and religion both deal with matters that are not strictly evidentiary.

      I can't say I agree with this hypothesis.

      The sciences (all sciences, including the most firm and certain) all rely on principles that they use in drawing conclusions. For instance, they all rely on principles of logic. You can't assert conclusions from experiments without using the principles of logic.

      Philosophy provides some of those principles. Indeed, "Philosophy of science" is (or ought to be) considered a discipline that is foundational to the sciences. (And the discipline of logic is, itself, foundational to the philosophy of science, so it too is foundational to science.) The fact that scientists often may be ignorant of the issues that trouble the prior disciplines doesn't mean that they are not USING the results of those disciplines, just as they might not bother to expend huge efforts on studying symbolic logic when they use principles of logic.

      And the notion that the discipline of the philosophy of science is "not evidentiary" and therefore something lesser than "science" is just a non-starter: like the discipline of logic, science simply cannot function without the results handed to it by sound philosophy of science, and its use of evidence relies on sound philosophy of science. If one wishes to take all of philosophy as diminished in standing because it is "non-evidentiary", then one diminishes science itself.

      Metaphysics, like logic, provides some of the underpinnings upon which the philosophy of science operates. You cannot have a sound philosophy of science with an unsound metaphysics in the background.

      Are religion and philosophy neighbors on the plane? It would seem so. They certainly are in university humanities departments, had you not noticed. And so, is religion philosophy? There is evidence of this. Belief, after Davidson, is a propositional attitude, and both religion and philosophy are propositional.

      The fact that universities place "religion" and "philosophy" in the same boat is evidence, rather, that universities are operating on a BAD set of underlying positions regarding thought, knowledge, etc. For example, they treat propositions that can't be proven and are held to be in contention to constitute "matters for 'belief' rather than matters for 'knowledge". But a proper understanding of metaphysics and knowledge allows for truths that can be known without proof, i.e. first principles that can be known DIRECTLY rather than via a proof: self-evident principles. The rejection of this category by moderns is a methodological mistake that causes the mistaken assumption that "philosophy" is not about things that can be known, but only believed.

      It would be just as coherent to say "science is propositional, and rests on the belief that we can rely on experimental evidence, therefore it is a belief system."

      Delete
    2. Well, I can't say I agree with many of the hypotheses swirling about the world of science AND philosophy these days. I do enjoy contributing, now and then. Some people know more than I. I do not mind. In your final comment regarding science, you used the word, coherent. I read it twice, just to be certain. Seems to me that, contextually, you meant to say incoherent. Is that right, or did I misunderstand?

      Delete
    3. Re coherence: I was providing a view of science that undermines science by calling it a "belief system". But its rationale is a bad one, and few (if anyone) actually holds this view of science. I was saying that the rationale works the same for both science and for philosophy, so if it is coherent for showing philosophy is a belief system, then it is also coherent for showing science is a belief system.
      In fact I would say that it is incoherent with respect to both, and neither philosophy nor science are properly called "belief systems".

      Delete
  11. Hi ccmnxc and The Deuce,

    For the record, I broke with the ID movement back in 2016, after concluding that its scientific claims didn't add up and that its mathematical arguments were invalid.

    The Thomistic version of hylomorphism espoused by Koons (and by Ed) is quite radical. What it says is that a proton which is part of a human body isn't a proton: the constituent particles of ordinary objects are "only potentially rather than actually entities in their own right," as Ed succinctly puts it in his review. The claim being made here is not just that they're only potentially independent; rather, what Thomists claim is that they're only potentially entities. However, Albert the Great and Duns Scotus held a different view: namely, that the human body contains forms at multiple levels, which means that the particles comprising it are still particles.

    A physicist will tell you that the protons in a human body are indistinguishable from free-floating protons, and that their properties undergo no change when that body dies. This indicates that they do not cease to be entities, when they're incorporated into a human body.

    I might add that there is a fundamental dis-analogy between quantum indeterminacy and the Aristotelian notion of potency. According to Aristotle, while an object is only potentially F, it is not actually F. By contrast, with quantum indeterminacy, when some property of a particle (say, its spin) is not defined (say, "up-or-down", or "F-or-G"), we cannot say it is not actually F ("up"); all we can say is that it is not (actually) up-and-not-down (F-and-not-G) or down-and not-up (G-and-not-F).

    The discovery of indefinite properties at the micro-level has no bearing on the question of whether we need to invoke the concept of potency to account for accidental or substantial change at the macro-level.

    Finally, while I agree with Ed that radical atomism and radical holism pose a threat to Christianity, I would suggest that the antidote is not radical hylomorphism, but a more nuanced view, which recognizes that ordinary-level may be viewed as fundamental from one perspective, as aggregates from another perspective, and as modifications of a larger whole from yet another perspective.

    I'd like to wish you and Ed (and Koons) a happy New Year.

    ReplyDelete
    Replies
    1. @ Vincent Torley,

      Particles that are bound are in a different state than "free floating" particles. In a living body the state of these particles is a complicated superposition of orthoganal functions of position and time. The eigenfunctions of their states are similarly local and complicated since potentials themselves vary in a complicated way inside a living body, and they certainly change when the body no longer functions, as in death, when neurons, muscle cells, etc., are no longer transmitting and conducting action potentials. Whether a physicist does or does not believe that there is such a thing as death, i.e. whether or not anything changes is not relevant to the truth ( it is absurd to think that nothing changes). Hylemorphism is not inconsistent with radical state change of the wave function of the entire body with death.

      Furthermore, the study of brain states with various types of probes disturbs the states and causes them to be different than they would be while private, according to quantum mechanics, so attempting to understand a free mind by probing brain states is bound to be beset with difficulties. Let's whack the golf ball off the tee and then study the tee to see what we can learn of the golf ball.

      :-)

      You seem to be involved in circular justification of your arbitrary prejudices here. I see no reason to take them seriously here.

      Happy New Year,

      Tom Cohoe

      Delete
    2. Let us see what say, Oderberg has to say about virtual existence. In pg. 71 of "Real Essentialism":

      "Supposing there to be elementary particles (a proposal I deny), and supposing these to be quarks, it does not follow from the fact that every material substance is made of quarks that every substantial form is the form of a bundle of quarks, because in the existing substance the quarks have no substantial identity of their own, their behaviour having been fully yoked to the function and operations of the substance in which they exist. The substantial forms of the particles exist virtually in the substances they constitute. In other words, the quark is ontologically dependent on the whole of which it is a part, but its causal powers persist, albeit in a way radically limited by the whole. The substantial form is what determines the permissible and impermissible behaviour of the quarks in the body, which is why some chemical reactions typically occur, others rarely, and others not at all. Nor is there any particular bundle of quarks of which the form could even be the form, given the familiar fact that every body loses and gains quarks all the time. Again, it is the form that determines the when, how, and how much of the loss and gain may occur, with external circumstances merely operating upon predetermined possibilities."

      From this characterization, what we should expect is that the behavior of a free electron (or whatever elementary particle) is radically different from an electron in an hydrogen atom (in a molecule, in a protein, in a cell, all the way up the levels of substantial organization), which is precisely what happens and any half-decent physicist will confirm. Or simply open a quantum mechanics textbook and see the radically different solutions of the Schroedinger equation for a free particle or a particle in a potential well or in a Coulomb potential, etc.

      "However, Albert the Great and Duns Scotus held a different view: namely, that the human body contains forms at multiple levels, which means that the particles comprising it are still particles."

      This is a related, but different issue (and indeed one hotly debated in medieval times), the unicity of substantial form.

      "By contrast, with quantum indeterminacy, when some property of a particle (say, its spin) is not defined (say, "up-or-down", or "F-or-G"), we cannot say it is not actually F ("up"); all we can say is that it is not (actually) up-and-not-down (F-and-not-G) or down-and not-up (G-and-not-F)."

      This is not what quantum indeterminacy, to call it that, means. At all.

      Delete
    3. Hi Tom Cohoe and grodrigues,

      We seem to be talking at cross-purposes. What I'm talking about are the intrinsic properties of elementary particles (such as quarks and electrons). These do not change when particles are bound in a structure - be it a living human body or an inanimate chemical lattice. What changes, as you rightly point out, are the interactions between the particles and their surrounding environment - interactions which can be described by the Schroedinger wave equation that governs their wavefunctions. (Yes, I have studied the Schroedinger wave equation, although it was over 40 years ago.) The fact that elementary particles behave differently when in the neighborhood of other particles no more proves that they cease to be elementary particles and that their identity is subsumed within that of the body they comprise than the fact that cars in a traffic jam behave differently than cars on the open road proves that they thereby cease to be cars and that their identity is taken over by that of the traffic jam they comprise.

      So when David Oderberg (who is a philosopher) writes that "[t]he substantial forms of the particles exist virtually in the substances they constitute. In other words, the quark is ontologically dependent on the whole of which it is a part, but its causal powers persist, albeit in a way radically limited by the whole," I have to respectfully disagree. A bound quark is still a quark, and its causal powers, which derive from its intrinsic properties, are still intact. Putting it another way: we don't need to do biology in order to understand quarks, be they quarks inside a human body or outside.

      Of course, as I mentioned earlier, particles themselves can be viewed as excitations of underlying fields, but that's another matter altogether, and one that's irrelevant to hylomorphism.

      Re quantum indeterminacy: pardon my somewhat clumsy formulation, but I was alluding to the fact that the propositional distributive law, p and (q or r) = (p and q) or (p and r), doesn't appear to hold in the quantum realm. To be fair, this claim is contested - see this paper here, which appeals to Bohmian mechanics (which is itself a highly contentious interpretation of QM, as it posits hidden variables): https://www.mdpi.com/2624-960X/2/4/42/htm

      My point was that whatever Aristotelian potency means, it certainly doesn't mean quantum fuzziness. Cheers.

      Delete
    4. (1) That “p and (q or r) = (p and q) or (p and r)” does not hold in the quantum real (is true when we consider non-commutable observable); but it does not hold in our macroscopic world neither also depending what you are observing, e.g. if you are measuring the probability of voting choice a sample of the population will express (just replace p, q and r by some political antagonists names in your part of the world). The logic formula “p and (q or r) = (p and q) or (p and r)” is always valid only in a realm where Boolean logic is always valid, e.g., Newtonian physics. Aristotelian logic is a generalization of that Logic, hence the reality’s structure (included the quantum’s one) it describes has not always abide to that.
      (2) As physicist we must not forget that QM as well its subsequent application in particle physics is a “just” a narrative which works (well, at least it is what Copenhagen’s school teaches us). As philosophers, we must give a hermeneutic, or if you prefer, a frame of interpretation for this narrative. Now, philosophically, we do not need a big bunch of new conceptual tools to describe what kind of categories of entities this narrative can be referred to: the Aristotelian one set is more than sufficient, not only at the level of the logic involved but also at the level of the concepts. We are surrounded by realities which are not individualized, not only quarks, electrons and fields of any kind, but also colors, warmth, affection, knowledge. We are also surrounded by beings which need to be “connected” to a bigger whole to exist as such, not only quarks, protons and electrons, atoms and crystals, but also limbs and bodies, leaves and trees, and so on and so forth. As philosophers we must understand that reasoning on a “free” electron is the same as reasoning about a “free” color which would wander by itself, about qualities and accidents: it will follow some rules of the logic and we can even affirm what happens when the yellow color “meets” an orange when this one fruit is ripe. Equations of movement “just” tell us what happens once one of these qualities meets the substance which will allow it to transition (projection in a Hilbert’s space) into something measurable, i.e., something actual, e.g., an “explosion” of any kind of particles (?) or, more precisely analogic and digital electronic signals, in the ATLAS instrumentation at CERN which confirms, or not, the … narrative.

      Delete
    5. @ Victor Torley,

      We are speaking at opposed purposes, not cross-purposes.

      :-)

      "the intrinsic properties of elementary particles (such as quarks and electrons) [...] do not change when particles are bound in a structure"

      In quantum mechanics the properties and existence of a particle is known only through observed interactions with other particles and these interactions are certainly different when the particle is bound then it is when it is free, so it is merely semantics whether you call it the same particle or a different particle that is found by probing. It definitely has different properties. "Intrinsic properties" are properties with reference to "self" and are not directly accessible but are inferred by extrinsic measurement, which changes, as I said, when the probe is into a bound system versus into relatively free particles. A free particle is _potentially_ a bound particle if it can be taken up into the bound system (see that word root "potential" there and think of Aquinas when you do).

      Another difficulty with your view is explaining how unique particles, like red blood cells (they are not identical), to which quantum mechanics also must apply, could be probed experimentally at all for fit with the laws of quantum mechanics. Red blood cells also exist in superpositions of states, but since each RBC is unique, there is no way to test its interactions for interference.

      I spoke of a golf ball being in a different state after being knocked from a tee. Note that the state, when it is flying, is one which it is only potentially in before it is hit. It certainly has different properties which it only potentially had before being sent flying. Get in the way of the flying ball. It hurts, but the ball sitting on the tee doesn't.

      Your propositional distributive law is not about intrinsic properties either. Particles behave. They do not do logic calculations with propositions. Your propositional distributive law is something that exists in our minds, not in particles. It is not relevant to Aristotelian action and potential, which fits very well with, i.e. does not contradict, quantum mechanics.

      Your argument simply fails to show otherwise.

      :-)

      Tom Cohoe

      Delete
    6. @Vincent Torley:

      "A bound quark is still a quark, and its causal powers, which derive from its intrinsic properties, are still intact. Putting it another way: we don't need to do biology in order to understand quarks, be they quarks inside a human body or outside."

      Bound particles behave radically differently than free particles, period, end of story. The fact that a bound electron (or quark or whatever) retains some of the properties (same mass, same spin, etc.) was never in dispute (that is, that in *some* sense, electrons remain electrons -- to quote Oderberg "The substantial forms of the particles exist virtually in the substances they constitute."), except maybe in your imagination. If you want to argue against the "radical" position of Profs. Feser and Oderberg philosophically, be my guest, but let us not pretend that physics gives you any special ammo.

      "quantum indeterminacy: pardon my somewhat clumsy formulation, but I was alluding to the fact that the propositional distributive law, p and (q or r) = (p and q) or (p and r), doesn't appear to hold in the quantum realm."

      Indeed, if you cook up the lattice of quantum propositions, it is not distributive (which is one of the reasons, why there are substantial difficulties in interpreting it as a logic). How that goes to show that "there is a fundamental dis-analogy between quantum indeterminacy and the Aristotelian notion of potency" is anyone's guess.

      "My point was that whatever Aristotelian potency means, it certainly doesn't mean quantum fuzziness."

      I will note that in the sentence quoted above, you speak about "analogy" and here below "means", two completely different things; I have not read Koons, but I suspect that you have no idea about what you are criticizing.

      Delete
    7. @ Vincent Torley,

      You may have missed a reply to you because I accidently addressed it to "Victor" Torley. It was posted on January 7th.

      Sorry about that.

      Tom Cohoe

      Delete
  12. Greetings from Brazil ! I have been an atheist and communist my whole youth and teen ages untill in 2017 i started to read conservatism and story of the church, cristianism etc (Olavo de Carvalho helped me a lot). Nowadays i reiceved many good reviews of your books, Mr Edward Feser. What book would you reccomend me to make me get rid once for all from this materialist thinking that is still left in my mind, i have been trying for so long to let my atheism go and dive deep into transcendency, thank you so much !

    ReplyDelete
    Replies
    1. @Rafeal: if diving deep into transcendency makes you feel good, then go for it. If you are applying the "outsider test for faith," then you might consider why Christianity, out of all the 'itys and 'isms, should be considered true, with the others considered false. If you're attracted to conservative politics, go for it, but be careful about concluding from that that Christianity, or Catholicism, is true and all other systems false.

      Delete
    2. I would be cautious of listening to the words of our Friend-Who-Won't-Pick-A-Screen-Name. It's great that you've started to come to discard materialism, as it's a dead-end philosophy that cannot satisfy the human soul, no matter what anyone says. My advice would be to immerse yourself in good philosophy, but also in a guided journey through Scripture, for instance, Fr. Mike Schmidt's "Bible In A Year" series. I would also encourage you to remember that years and years of thinking are not easily discarded, so don't get discouraged if you find it difficult to stop thinking like a materialist right away. It will happen gradually, by the Grace of God, who loves you.

      Amen.

      Delete
    3. I also concur with the advice of another anonymous poster below, who recommends prayer - always a good thing.

      Delete
    4. Hey @Rafa,

      I'm Brazilian too so I think I can help you with this. First, I must say that Aristotelian-Thomistic metaphysics (A-T for short now on) is well-developed and could be hard to understand at first (and indeed, it could take a long time to get a lot of it right) but Ed's turn makes all of that intelligible and very readable for lay people like us. Second, if you grow up like me (I'm from around Sampa, btw) and had that *amazing* educational system regarding philosophy and related matters, I friendly suggest that you get rid of all of these crap notions that you could have learned about the soul, religion and etc. because this will effectively hinder your knowledge - and you will be reading into A-T metaphysics something that's not true of it!

      For a start, I do recommend the famous The Last Superstition (TLS, for short), but I urge you to not buy the Portuguese version (even though the cover art is awesome), but instead, you should buy the yellow cape OG version. I know it may sound paradoxical, but you will understand the English version much better than the Portuguese one because our Portuguese version looks like the people who translated it have a superficial understanding of the doctrine and even less knew Ed's amazing style of writing a book (i.e the Portuguese version is a pedantic, hard to read and misleading just like the great majority of continental philosophy books that we are familiar with). I don't know if you share that opinion about our philosophical school (continental) but I can say without thinking twice that I only could grasp things truly when I read the original version. So keep that in mind.

      I do recommend though that you read the book calmly, and carefully and not read your own understanding into the book itself. And when you get a better familiarity with A-T (through the monumental, careful, and well-detailed blog posts that Ed wrote throughout all these years and the TLS itself) I do recommend, with some cautions, Scholastic Metaphysics: A contemporary introduction. The caution part is because, even though Ed's incomparable ability to make it readable through and through you gonna face some things - to say the truth a lot of it - that we are unfamiliar with because of our traditional philosophical school (e.g causal powers, dispositions, modality, essence and existence, identity over time in some sense, and others).

      If you want to know more about Our Father, I do recommend you to read the wonderful Five Proofs for the existence of God. In this book, Ed goes through five philosophical arguments given for the existence of God. In special, it would be nice if you already have some familiarity with the Act/Potency distinction - even though Ed goes through to explain it in the book - because if you grasp that distinction, you will see why the Aristotelian Proof further developed by Saint Thomas must necessarily follow from the philosophical premises of the argument. You don't need faith to know that God does exist, but after knowing that His existence sure follows and that for some reason does not change your faith... I dare to say that you don't understand the argument right.

      And, of course, there are A LOT of good-willed fellows out here in the blog (e.g Michael Copas, grodriges, Mister Geocon, UncommonDescent, and my homeboy Talmid just to say a few ones that I remember). Don't worry, I know that the world around us is full of very bad people, but in here you could feel at home!

      Deus te abençoe, cara!

      Delete
    5. Being summoned by my boy Tadeo, here i'am!

      @Rafael Nobre

      Besides the very helpful tips you received, i add more two:

      1. I have a little blog myself were i did write on some of these things, enjoy: https://meioquedialetica.blogspot.com/?m=1

      2022 was quite a busy year, but i plan to update the blog again on this january.

      2. Reading Olavo, take a look at his Consciência da Imortalidade. I heard it is a good take on the soul, but i never read it myself, i'am starting the guy by another path.

      This is to complement the reading recommendations. I also advice you to look very well on these readings not as way to refute materialists, but as just ways of knowing the truth, agreeing or not. Do not be fast to change your ideas, even if i happen to think that these guys are right i say that, but look at the arguments and what they say about YOU and your life. Truth must be relevant to the individual, something that one understands when going on the christian life, which i hope you do someday.

      And a final tip that is a complement: do not stop at reading, try to see how you are living and what is the philosophy that it says, the identity it means. I think that Olavo talks about that(his examination of consciousness/exame de consciência).

      Busque a verdade e sei que A Verdade você verá que estava o tempo todo em teu interior, amigo!

      Delete
    6. @ Cantus, I'm the Anonymous who didn't post a screen name because I was having trouble with the comment screen, lol.

      You wrote this: "It's great that you've started to come to discard materialism, as it's a dead-end philosophy that cannot satisfy the human soul, no matter what anyone says."

      Rafael Nobre may well be on a better track if he adopts a position other than materialism. I think materialism has a problem in denying that abstract objects exist, for example.

      But can you unpack what you have in mind when you speak as though a reason to discard materialism is that it "cannot satisfy the human soul"? Even your wording raises questions, for what do you mean by "soul"? Are you counseling Rafael to adopt a philosophical system because it may "satisfy"? There are many systems that may make us feel fulfilled or the like (not sure yet what you consider "satisfying") -- e.g. maybe someone ELSE's conception of communism -- but I would have thought that we are justified in adopting a system only if we are convinced that at least it's highly likely to be true. If we don't know, I should think we are epistemically bound to suspend judgment.

      Delete
    7. Are you counseling Rafael to adopt a philosophical system because it may "satisfy"?

      Humans desire to know, it's built into human nature. Not knowing anything is inherently unsatisfying.

      Materialism denies the possibility of knowing, because it reduces all the activity we otherwise think of as parts of the process of knowing as being merely material activity, and merely material activity cannot constitute knowing. An atom emitting a photon or a neuron firing upon receiving a physical stimulus cannot mean some known proposition in a way that correlates to actual truth, (i.e. that the photon or neuron action does so BECAUSE the proposition is true and is held in the knower as true on account of some logically adequate ground).

      Perhaps we shouldn't hope to know things if it is impossible to know things; but if materialism is true, it is impossible to know that it is true and impossible to know that it is impossible to know things - it is a self-defeating hypothesis for a being that is trying to know.

      Delete
    8. @Tony: your answer on Cantus' behalf does not answer the questions I asked. I did not ask for a refutation of materialism; I agreed that materialism has problems.

      Cantus was replying to Rafael Nobre, who had asked for reasons to drop his materialism and atheism and "dive deep into transcendency". Cantus told Rafael that materialism is a dead-end philosophy that cannot satisfy the human soul. I am still waiting for Cantus to show why ability to "satisfy the human soul" provides SUFFICIENT reason for us to adopt a philosophy of "transcendency". Rafael seemed to be conflating atheism with materialism and to incline to think that maybe Catholicism is the way to go--at least, he seems to think that dropping materialism will land him in Catholicism.

      But since there can be a number of philosophies or systems that "satisfy," it is not clear which one is true, if any are true, from an ability to "satisfy."

      I'll note by the way that you seem to narrow materialism down to its "eliminativist" versions. To point out inconsistencies in a given materialist's discourse does not by itself falsity materialism. That's the problem with retorsion arguments; they usually do not falsify the thesis at which they're aimed, although they can succeed in showing that a proponent of a thesis is being inconsistent.

      Delete
  13. Rafael
    You don't need to read a book to rid yourself of atheism and materialism. You need to pray.

    ReplyDelete
  14. I looked back at the Koonsian title, a question on obsolescence. Seems to me that, sooner or later, everything/everyone becomes obsolete. This is not rocket science. Which is approximately, if not exactly, the point. It matters not to me how others cleave to the history of religion, philosophy or science. Those cleaveages (chuckle) are based on interest, preference and motive. If, and only if, this is a foundation of his thesis, it is tired rhetoric. Read Kenneth Burke if you care to know about rhetoric.

    The ideas, manifestos, theses of many thinkers are obsolete. If science, as some claim, is in trouble; civilization is in triple trouble. Faith, as a practical influence, is worthless. Metaphysics is of little value.
    A pity. It was an origin of science. Hmmph, said the camel...

    ReplyDelete
  15. from Prof. Feser's review: 'According to hylomorphism, the parts of a material thing are less fundamental to it than the whole—and, indeed, exist in the whole only as potentially independent things rather than as actual entities in their own right. For example, hydrogen and oxygen exist in water, but only as constituents of the water rather than as entities in their own right. They potentially have such independent existence (via electrolysis), but until that happens what actually exists is the water as a whole substance."

    1. As I remember, in Aristotle, a part of a substance can itself be a substance. An eye is a substance; "pale" or whatever is said "of" the eye, i.e. the eye has accidents.

    2. The four "elements" are archai, "first principles," according to De Generatione et Corruptione. But archai are causes. And ontologically prior to the four "elements" are the contraries, hot/cold, wet/dry. What is meant by saying, e.g., that the water and the wet/cold exists only potentially in, say, a frog, only to be made actual when the form-matter composite is granted an act of existence, if "water" and "wet/cold" are archai of the frog?

    ReplyDelete
  16. I tried to comment but can't figure out what seems to be a new system. I'm too old to keep up with this blog, lol.

    ReplyDelete
    Replies
    1. Which "new system"?
      I have no problem replying (if this reply shows up, that is).

      Delete
  17. Back a ways, (January 5) I remarked that I did not think much of either Aristotle or Aquinas as physicists. That drew criticism of course. I also commented on obsolescence, at one point. My remarks appear to have gotten some agreement from Stardust, who sounds well-read and a lucid thinker. Obsolescence is, at times, relative, at times tied with irrelevance. The thinkers referenced were not obsolete in their time(s). No one knew any better, or if they did, were not talking; or, if talking, were not being heard. If Albert Einstein had spoken with either figure, he would have both smiled and frowned at their explanations/exclamations. Other more recent thinkers likely would have reacted in like manner. Responsible people, IMHO, try harder to think better and do the best they can with what they have and know. That is all any of us can do.

    ReplyDelete
  18. I posted the stuff above about GC. The standard view about elements in Aristotle is this:
    ". For Aristotle an element of something is the most basic constituent of that thing. It is indivisible into
    things that are different in form, which is to say that it cannot be analysed into further constituents - or 'elements' - of its own. The elements of bodies, then, are the simplest constituents of bodies (Metaph. V.3, 10l4a26-34, De cáelo III. 3, 302al4-21). ... Aristotle analyses fire, air, water, and earth into more fundamental constituents. Consequently the latter - usually believed to be the primary
    'qualities', the contraries hot and cold, dry and wet, and often also 'prime
    matter' - are more deserving of the name στοιχεία, or 'elements'. These items - the contraries alone, or together with prime matter - are considered the true elements of bodies because together they compose fire, air, water, and earth... in his view, fire, air, water, and earth are not elements strictly speaking, because they reveal under analysis further, more fundamental, that is, more elemental, items." ~ Timothy J. Crowley, Phronesis 2008.

    In the Meteorologica, Ari says that what we call fire is different from "real fire," which is an excess of heat. The problem I'm puzzling over - and not just now, though Prof. Feser's review recalled it - is what happens when, say, a hot thing is destroyed. "The hot" in it doesn't just poof out of actuality and revert to prime matter. Something hot is left over and is a constituent of a new thing. So it's a puzzle, how "the hot" would NOT be a constituent of either the prior or the subsequent heated substance.

    This question recurs when we substitute protons and neutrons or whatever for "the hot/cold/wet/dry." For Aristotle, "an 'element', strictly speaking, is the primary material constituent into which bodies divide, or into which bodies can be analysed {Metaph. V.3, 10l4a31-34; De cáelo III.3, 302al4-19; cf.
    Metaph. III.3, 998a22-b4; cf. Metaph. VII. 17, 104lb31: στοιχεῖον δ' εστίν εἰς ὃ διαιρεῖται ἐνυπάρχον ὡς ὕλη)" ~ Crowley.

    One may say, we never have "the hot" just by itself apart from substances that are hot. On the other hand, "the hot" bridges across corruption and generation, so it's hard to see how it exists only potentially apart from either substance. It does work during the bridge, so it seems to persist in act. Lindsay Judson said in a colloquium that what persists from corruption to generation is just the hot's "extension." Unfortunately, I never got the chance to ask him to develop this thought afterwards.

    ReplyDelete
    Replies
    1. ficino,
      "Lindsay Judson said in a colloquium that what persists from corruption to generation is just the hot's "extension." Unfortunately, I never got the chance to ask him to develop this thought afterwards."
      I am having a bit of difficulty figuring out if this was a serious first person post, or are you just trying to sort out the details of all those ancient misconceptions for historical research purposes? I hope the latter.

      Heat is kinetic energy of motion at the particle/atomic scale/molecular scale. Temperature is direct function of the average velocity of such particles.

      Aggregate thermal "laws" were worked out centuries ago prior to modern physics. QM provided a theory of, for example, blackbody radiation, which is related to the term "color temperature".

      When a moving object, say, a billiard ball, slows and stops it's motion is not lost, as Aristotle supposed, rather, it's kinetic energy of motion is transferred to kinetic energy of the surrounding environment, some of which goes to heating up the surrounding air. That effect is too small and too diffuse to be easily detected without instruments, so Aristotle did not realize this fact.

      Using Aristotelian terminology and concepts for heat is hopeless, he got it wrong. I suggest you consult any university physics book, or research the subject on line using modern physics educational resources.

      Delete
    2. @Stardusty:

      "I am having a bit of difficulty figuring out if this was a serious first person post, or are you just trying to sort out the details of all those ancient misconceptions for historical research purposes? I hope the latter."

      The latter. But the questions are relevant to the OP because Prof. Feser and Koons are working from conceptions that have their root in Aristotle. I am both puzzling over Aristotle and am questioning what I take to be Feser's contention that Aristotle denies that natural substances have ACTUAL elemental constituents.

      Delete
  19. Some of the commenters to this post are pretty mired in the 'perfections' of physics in modern times. I understand that, I think. Insofar as claims regarding fusion energy; artificial intelligence; transhumanism and machine consciousness are not doing so well, one wonders what the protagonists have to crow about? This stupid machine I am using does not know the difference between 'crow' and 'grow'. Transhumanism got translated as 'grandbabies'. Really? Does anyone think AI can approach the nuances of human reason? In any coherent way. If there is no reliable means of teaching machinery human cognition, what good is it, beyond building automobiles or somethings else, on an assembly line? Well, yes. Economically speaking, an employer does not need to worry about health insurance, or retirement benefits. Trade offs go to capitalism---our bread and butter. When machines wear out, they are scrap. Fair enough.Everything blows up; breaks down; falls apart or wears out. Sellars quipped: philosophy is an attempt to see how things...hang together...Nagle said: reality is how things probably are; not how they might possibly be. Rorty said pragmatism has consequences. Yes, it does

    ReplyDelete
  20. In Feser's review at Public Discourse he states that the "common sense" view of QM is "basically right". Given that we know the failures of "common sense" views in math and physics, why should anyone take "common sense" as a reliable metric to the workings of the universe? Feynman, for example, in https://www.youtube.com/watch?v=41Jc75tQcB0, says:
    "We see things that are far from what we would guess. We see things that are very far from what we could have imagined and so our imagination is stretched to the utmost … just to comprehend the things that are there. [Nature behaves] in a way like nothing you have ever seen before. … But how can it be like that? Which really is a reflection of an uncontrolled but I say utterly vain desire to see it in terms of some analogy with something familiar… I think I can safely say that nobody understands Quantum Mechanics… Nobody knows how it can be like that."

    ReplyDelete
  21. Would a relative layman in both hylomorphism and quantum mechanics be able to profit from this book? I've always been a big fan of Koons, but a lot of his stuff is a bit over my head.

    ReplyDelete
  22. ..."nature behaves in a way like nothing we have ever seen before...' Well of course it does. That is the fundamental point of what we call science. Richard Feynman was one of half a dozen geniuses of the twentieth century. Along with Turing, Russell and Oppenheimer. The amazing thing about geniuses is they know their limitations. I admire Dr. Feser for sticking to his convictions. I do not have to agree with them. I have asserted that metaphysics is a tool for thinking.. not an end within itself. I stand by that.
    As someone dear to me has said:
    Kirk, out.

    ReplyDelete
  23. Given that we know the failures of "common sense" views in math and physics, why should anyone take "common sense" as a reliable metric to the workings of the universe?

    Depends on what one means by "common sense" doesn't it? It's common sense that one's conclusions should not contradict one's premises nor one's method for reaching one's conclusions.

    Take for instance the Copenhagen interpretation of QM. Nothing really exists until it is measured. But the measurement instrument/observer must already exist in order to measure anything... creating a vicious regress.

    It's what happens when scientists are poor at philosophy.

    ReplyDelete
    Replies
    1. The state of a particle does not exist until it is measured; but the measuring apparatus does ( because it has been measured). Take Copenhagen to mean that the future does not physically exist until you encounter it. This has been experimentally verified. Furthermore, take a look at the delayed choice quantum eraser experiment - where something that happens after a photon hits a detector affects what is recorded by the detector. Philosophically, how can that be?

      This is what happens when philosophers are poor at science.

      Delete
    2. The state of a particle does not exist until it is measured; but the measuring apparatus does ( because it has been measured).

      What does it mean to say that "the state" of a particle doesn't exist? The particle exists but is "stateless"? Wouldn't an existing particle have to be in some sort of state or another?

      Next. What measured the "measuring apparatus" that has been measured? What measured that? And so on? This is the vicious regress I referred to.

      Delete
    3. "Nothing really exists until it is measured. But the measurement instrument/observer must already exist in order to measure anything... creating a vicious regress" This is not what Copenaghen's school states.
      The fact to pretend that a property must be existing independently from the measurement is a typical Newtonian property, called determinism, which is needed to constrain the lattice of propositions of the classical physics along a Boolean logic.
      This determinism is not needed neither in our current world nor in the Aristotelian physics, nor in Quantum Mechanics. Causality is needed but not determinism: their logic's structure do not need this extra assumption.

      Delete
    4. A measuring apparatus can only measure what is actual never what is potential.
      Measuring a potentiality is meaningless: one has to have to move the potency into actuality, which means that the object measured is not precisely the same as the object before the measurement, in the sense that it will have lost a potency and acquired a new actuality (and probably lost some of the previously actual characteristics).
      This is true when I consider if this chalk is breakable ( the only way is to break it) or if I want to measure if that electron is in a state up and by doing that projecting (making actual) his initial state into that state up.

      Delete
    5. bmiller: What does it mean to say that "the state" of a particle doesn't exist?

      See Is the moon there when nobody looks? Reality and the quantum theory

      What measured the "measuring apparatus"

      You did. Only you experience the future becoming now for yourself.

      Delete
    6. wrf3,

      Regarding "stateless"
      I didn't find the answer to my question in the physics paper. Do you have one?

      Regarding "measuring apparatus":
      I've been called many things, but "measuring apparatus" is a new one. Before there were people with particle accelerators, where there dinosaurs with particle accelerators that created the present?

      Delete
    7. It's right there under the sub-heading "A Fact is Found" and the 3rd paragraph ("Because there are no built-in ...") under "A gedanken demonstration".

      Observers do not create the outcome of measurements. They simply gain knowledge they otherwise would not have had.

      Delete
    8. Your latest statement seems different from this:
      The state of a particle does not exist until it is measured

      Knowledge of the state that a particle is in is different from the actual state a particle is in. Knowledge of the state is epistemological and the actual state is ontological. Your original statement implied, to me, an ontological statement but your latest statement implies you were referring to the epistemological state which I agree with. Perhaps you can see why saying "The state of a particle" rather than the "The knowledge of the state of a particle" would lead me to believe me to think you were talking about ontology rather than epistemology.

      In fact, that is part of the problem of the literature on QM. The confusion of the actual with the knowledge of the actual. It seems this is an obvious confusion, so why don't people who know better point it out?

      I agree with Feynman (from the article)

      You know how it always is,
      every new idea,
      it takes a generation or two
      until it becomes obvious
      that there’s no real problem.


      "common sense" used to be opposed to the thought that earth rotated because things would fly off the surface of the earth like mud from a bicycle tire until there was enough incentive to investigate more deeply and discover that the mass of the earth allowed for that. Give this time. However, I think we are becoming dumber in the near term.

      Delete
    9. My reply to this seems to violate some "internal rule". Will try again

      Delete
    10. Your latest statement seems different from this:
      The state of a particle does not exist until it is measured

      Knowledge of the state that a particle is in is different from the actual state a particle is in. Knowledge of the state is epistemological and the actual state is ontological. Your original statement implied, to me, an ontological statement but your latest statement implies you were referring to the epistemological state which I agree with. Perhaps you can see why saying "The state of a particle" rather than the "The knowledge of the state of a particle" would lead me to believe me to think you were talking about ontology rather than epistemology.

      In fact, that is part of the problem of the literature on QM. The confusion of the actual with the knowledge of the actual. It seems this is an obvious confusion, so why don't people who know better point it out?

      I agree with Feynman (from the article)

      You know how it always is,
      every new idea,
      it takes a generation or two
      until it becomes obvious
      that there’s no real problem.


      "common sense" used to be opposed to the thought that earth rotated because things would fly off the surface of the earth like mud from a bicycle tire until there was enough incentive to investigate more deeply and discover that the mass of the earth allowed for that. Give this time. However, I think we are becoming dumber in the near term.

      Delete
    11. Sorry about all the postings. Things seem to have got bottled up and then let loose at once.

      Delete
    12. bmiller: Your latest statement seems different from this: "The state of a particle does not exist until it is measured."

      I assume you're referring to "an observer does not create the state of the particle..."

      The two are not in conflict. The state of the particle does not exist until it is measured, and it isn't the measurement that creates the state.

      Read the section "how could it do it" in Mermin's article.

      Delete
    13. bmiller: "common sense" used to be opposed...

      You just agreed with my statement that "common sense" simply isn't a reliable metric for understanding nature.

      Delete
    14. wrf3,

      You just agreed with my statement that "common sense" simply isn't a reliable metric for understanding nature.

      Not necessarily. Like I mentioned before. It depends on what you mean by "common sense".

      Do you think people who don't have common sense are reliable guides to understanding anything?

      Delete
    15. wrf3,

      The two are not in conflict. The state of the particle does not exist until it is measured, and it isn't the measurement that creates the state.

      What creates the existent state from the non-existent state then?

      This quote from the article seems to disagree with your assumption that the measurement doesn't create the state.

      “Observations not only disturb what has to be measured, they produce it….We compel [the electron] to assume a definite position…. We ourselves produce the results of measurements.”

      Jordan’s statement is something of a truism for contemporary physicists.

      Delete
    16. bmiller: It depends on what you mean by "common sense".

      That's the problem, isn't it? It doesn't come in units that can be agreed upon. How person A thinks things are isn't how person B thinks things are isn't how person C thinks things are. It simply isn't, nor has it ever been, a reliable guide to understanding nature.

      Delete
    17. bmiller: What creates the existent state from the non-existent state then?
      Nobody knows. All we know is that future things do not physically exist until they are experienced, i.e. when the future becomes now.

      Measurement doesn't cause the result of the measurement, that is, something isn't spin up because it was measured. It was measured and found to be spin up.

      Delete
    18. wrf,

      That's the problem, isn't it?

      People disagree. It's a not a problem confined to "common sense" as demonstrated by the article you linked to.

      Delete
    19. wrf3,

      Nobody knows. All we know is that future things do not physically exist until they are experienced, i.e. when the future becomes now.

      Are we talking past each other?

      Certain interpretations of QM claim that presently existing things have no "real" state due to superposition. Like Schrödinger's cat.

      Is the cat neither dead nor alive if no one looks?

      Delete
    20. bmiller: Are we talking past each other?
      Probably. You're trying to use your "classical" (as in classical physics) "common sense" to try to wrap your head around something that simply isn't classical. Your common sense is failing you. I suspect that you're confusing the quantum description of an entangled system with the actual physical state of the system. Quantum descriptions, being probabilistic, describe what might be. What might be is future. What is future does not physically exist.

      Did you look at the delayed choice quantum eraser experiment? Do you get how it turns everything you think you know upside down?

      Delete
    21. wrf3,

      Thanks for sharing your guesses. But I don't recognize what I wrote from that. I must really be a bad communicator.

      From your answers I can't even guess what you think. It seems to me that you are confusing determinism with reality much like how Bohr didn't understand what Einstein was getting at in their argument.

      Yes, I know that QM doesn't offer deterministic answers to future predictions but so what? Are you saying that some people think the future exists in the present? Who?

      It seems to me that you've been arguing that the present doesn't even exist in the present (unless someone is "measuring" it).

      Help me out. Schrödinger's cat. Is the cat neither dead nor alive if no one looks?

      Delete
    22. bmiller: From your answers I can't even guess what you think
      That's because you're trying to understand the quantum way of looking at things by the classical way of looking at things. Can't be done. That's why Bohr understood Einstein, but Einstein didn't understand Bohr.

      QM does more than not offer deterministic answers to future predictions. It says that the future does not physically exist.

      Think about your statement "the present doesn't exist in the present". What, exactly, is the present? If you go to measure something, then what you're measuring isn't in the present - until you measure it.

      I'll help you out with Schrödinger's cat, but not before you're ready to understand it. You're still asking a classical question about quantum events. First, tell me what it happening with the delayed choice quantum eraser. How does something that happens later in the experiment affect something that happened earlier in the experiment? If you aren't going to deal with that, you aren't ready to think about the cat.


      Delete
    23. wrf3,

      Maybe I'm wrong about who is a poor communicator.

      Who ever said the future "physically exists"? I certainly didn't.

      What, exactly, is the present?
      Why don't you tell me what you think it is. And while you're at it define what you mean by "measurement".

      Look. If you can't explain your position just say so instead of giving me reading assignments that don't actually support your position. Honestly, I don't think you've read them yourself.

      Delete
    24. bmiller: Honestly, I don't think you've read them yourself.
      You just ended the conversation.

      Delete
    25. wrf3,
      You just ended the conversation.

      Thanks.

      Delete
    26. It was very difficult to get wrf3 to actually state what he thought. He kept claiming that the "Quantum Eraser" would blow our minds and finally stated that that it was a time machine:
      How does something that happens later in the experiment affect something that happened earlier in the experiment?

      This is not the case and a simple Google searche would show this is not a scientific conclusion:

      Common misconception
      https://en.wikipedia.org/wiki/Quantum_eraser_experiment

      Consensus: no retrocausality
      https://en.wikipedia.org/wiki/Delayed-choice_quantum_eraser

      Here's a nice video explaining it
      https://youtu.be/RQv5CVELG3U

      And Sean Carroll
      https://www.preposterousuniverse.com/blog/2019/09/21/the-notorious-delayed-choice-quantum-eraser/

      Delete
  24. Where does Heisenberg interpret QM as vindicating hylomorphism? I'd love to read that.

    ReplyDelete
    Replies
    1. https://edwardfeser.blogspot.com/2009/09/heisenberg-on-act-and-potency.html

      Delete
    2. The mass of bound particles such as electrons and protons is less than the mass of their free counterparts. Thus a hydrogen atom has a smaller mass than the sum of the masses of a free electron and a free proton. This lost mass in the hydrogen atom is called the "binding energy" of the electron and the proton in the hydrogen atom.

      If the masses of these particles are not less when they are bound then when they are free, then how could the total mass of the hydrogen atom be less than the sum of the individual free particles that make it up?

      Some people posting here seem to think that such particles "retain their identifying properties" whether they are free or bound into a combination that makes up something like a crystal, a metal, a brain, or a red blood cell.

      They do not as even such fundamental properties as mass change.

      This is inconsistent with their claims, but is completely consistent with hylomorphism, in which a free electron has the potential to become something different, such as a bound electron, when it becomes active as part of a living body, and then again as it becomes part of a dead body - at least that is what the fundamental property, mass, tells us.

      Tom Cohoe

      Delete
    3. Tom,
      If you change weight are you still you? Do you "retain your identifying properties" when a gram of water evaporates from your skin?

      Indeed, are you precisely the same at all moment to moment? Clearly, no.

      Is one apple precisely the same as another apple? Of course not, so, in that case, what sense does it make to count apples, since no two apples are precisely the same?

      The identification of a sort of thing is always a bit fuzzy, so we approximate to classify and consider certain sorts of objects to be members of a set when their "identifying properties" fall within some range established by convention.

      In chemistry the convention is that the number of protons in the nucleus determines which element the atom is called. Atoms of the same sort can vary over a wide variety of parameters, even the number of neutrons in the nucleus, and they are still called, by convention, a particular element name.

      Hylomorphism: stuff can change but stuff can only change into stuff that it can change to and it can't change to stuff it can't change to. This is what Thomists consider some grand profound insight?

      Such trivial, tautological truths are what remain of A-T in modern science. Oh yes, and Aristotle observed that stuff moves, so did Aquinas, so in that respect, yes, modern science has confirmed A-T.

      However, the rest of A-T is bunk because Aristotle got the fundamentals wrong. The primary argument for a first mover is unsound because Aristotle was fundamentally wrong about motion (change). That argument is also logically invalid as an argument for god, very clearly because Aquinas abandoned valid logic in the last sentence.

      Delete
    4. In what way was Aristotle wrong about motion?

      Delete
    5. @ StardustyPsyche,

      "Do you "retain your identifying properties" when a gram of water evaporates from your skin?"

      "Retaining identifying properties" is what I was saying others seem to think, i.e., they might think that a bound proton has the same mass as a free proton. I don't see how your question relates to that except, possibly, as rhetoric that misses the point.

      "However, the rest of A-T is bunk because Aristotle got the fundamentals wrong."

      You haven't debunked him.

      "That argument is also logically invalid as an argument for god, very clearly because Aquinas abandoned valid logic in the last sentence."

      Prejudice, which you express, is logically invalid as an argument. Circularity does not lead to anything except what you assume.

      :-)

      Tom Cohoe

      Delete
    6. …This character of the quantum theory already makes it difficult to follow wholly the program of materialistic philosophy and to describe the smallest particles of matter, the elementary particles, as the true reality. In the light of the quantum theory these elementary particles are no longer real in the same sense as the objects of daily life, trees or stones, but appear as abstractions derived from the real material of observation in the true sense. But if it becomes impossible to attribute to the elementary particles this existence in the truest sense, it becomes more difficult to consider matter as “the truly real”…

      Werner Heisenberg, On Modern Physics, p.13.

      Delete
    7. Perhaps even the most brilliant minds are confused, when it comes to theory and reality? I do not have that impediment. If it is one. And, of course, it is, when talking about physics, math and other constructs as opposed to the "reality" of the chair, upon which I now sit. I cannot think of my ass sitting upon an atom.

      Heisenberg, the genius, forsook that for a moment in questioning reality, even so far as he knew that neither he, nor anyone else could sit on an atom.it takes many. thanks, JD.

      Delete
    8. SeanJanuary 11, 2023 at 10:31 AM
      "In what way was Aristotle wrong about motion?"
      Aristotle was fundamentally wrong in asserting that sublunary motion is in an impeding medium such that a moving object requires a mover to account for its motion.

      This fundamental error of Aristotle is repeated by Aquinas in the First Way in the example of the staff and hand, wherein Aquinas asserts that the staff moves "only" because it is moved by the hand, which is clearly false.

      This fundamental error of Aristotle, repeated by Aquinas, leads to the unsound argument for the first mover.

      The truth is that all motion is in space. You are in space. We are all in space. Motion is never lost, only transferred or transformed. An object once set in motion will continue moving ad infinitum unless its motion is transferred or transformed.

      Aristotle had the notion of ad infinitum motion in his grasp, as expressed in his Physics book 4, but he discounted the notion of the void, thinking, among other things, that speed would go to infinity absent the sublunary impeding medium.

      Aristotle was not an atomist, and thus had no understanding that heat and temperature are functions of molecular or atomic motion, which progresses losslessly in space, thus negating the argument from motion for a first mover.

      Delete
    9. Tom Cohoe
      "Prejudice, which you express"
      Really? What have I expressed that seems to you to be based on prejudice?

      As an argument for a first mover the First Way is unsound because it is based on the false premise that an object observed to be in motion in the present moment must have an external mover in the present moment to account for its continued motion.

      As an argument for the existence of god the First Way is logically invalid because it employs the logical fallacy of non-sequitur in the last sentence, as well as fallaciously equating human understanding with logical necessity.

      Francisco J. Romero Carrasquillo put significant care into analysis and expression of the Five Ways of Aquinas in syllogistic form and symbolic format. The fact that the First Way is logically invalid as an argument for god is evidenced by the glaring omission of "and this everyone understands to be God" expressed in syllogistic form or symbolic format by Carrasquillo.

      The reason Carrasquillo omits any mention of god in the syllogistic form and symbolic format is obvious, such mention is logically invalid. There is no logically valid way to connect "and this everyone understands to be God" with the preceding argument.
      iteadthomam.blogspot.com/2011/01/first-way-in-syllogistic-form.html

      Hardly a matter of "prejudice" on my part, rather, elementary logical analysis. The First Way is incontrovertibly logically invalid as an argument for the existence of god.

      Delete
    10. @ StardustyPsyche,

      Your whole stance is based on prejudice.

      "elementary logical analysis"

      You don't even seem to understand what logic is, nor do you understand what Aquinas and Aristotle said. You seem to think that logic is some absolute supporting your prejudice rather than:

      1. - something that is argued about to this day and that becomes more complex continuously, and

      2. - something invented for proper human use as an exemplary divine idea.

      All bodies in temporal motion radiate energy as multi polar radiators and lose orbital motion, thus energy, and come to a natural state of rest. Nothing you say demonstrates that, should an orbit be sustained, it could not be by a prime mover. There is no inconsistency.

      As for "Carrasquillo", I don't much care what he says.

      "Aristotle was fundamentally wrong in asserting that sublunary motion is in an impeding medium"

      It _is_ in an impeding medium.

      "he discounted the notion of the void, thinking, among other things, that speed would go to infinity absent the sublunary impeding medium"

      Absent the impeding medium which removes energy from objects in motion, a continuing force would cause the speed to go to infinity. According to special relativity, the distance between the body moved and things which it approaches in the direction of motion grows to zero as the limiting velocity of light is approached. In fact, the first evidence that neutrinos had rest mass was that they changed type as they passed through the Earth, indicating that they did not "see" 0 distance in the diameter of the Earth.

      These things were not understood by Aristotle and Aquinas as some "magical" prophetic power about future science. The reasoning was from a completely different direction and that their direction of thought led to true natural theology is strongly indicated by modern science's actual non-contradiction of their results.
      I am sorry, but you do not seem to understand what their results say.

      Things that we have long understood do not "go away" just because prejudiced people say they have.

      :-)

      Tom Cohoe

      Delete
    11. Tom,
      "You seem to think that logic is some absolute supporting your prejudice"
      Logic is a human convention, to which Aristotle was an early and major contributor.
      Aquinas clearly valued logic and put a great deal of effort into an attempt at using logic to make a positive case for the necessity of a first mover, and therefore god.

      Dr. Feser clearly values logic, so much so that some of his posts are deep dives into very finely specified nuances of logical minutia, which I greatly appreciate. He and I agree on little, but if we can both agree on a common language of logical communication then at least we have a common basis for constructive engagement.

      If you do not value logic then there is no logical argument I can make that will even potentially form a basis for mutual understanding between us.

      "All bodies in temporal motion radiate energy as multi polar radiators and lose orbital motion, thus energy, and come to a natural state of rest."
      That is a version of Aristotelian misunderstanding, that when an object loses energy to an impeding medium and comes to a rest its motion is lost, and therefore, it is erroneously considered, given the fact that we observe motion generally, there must be a prime mover to keep it all moving.

      The error Aristotle, Aquinas, Feser, you, and most of the good posters here make is that the energy lost from the slowing and stopping object is not lost in total, rather, the energy of motion is is conserved and is merely transferred.

      For every object that slows and stops there is an equal energy of motion gained by other objects due to the conservation of energy. You may observe that an object comes to a rest, but what you are failing to realize is that other objects increase in their velocity by an amount equal to the observed loss.

      This forms a system of universal perpetual motion, making a first mover unnecessary.

      "Nothing you say demonstrates that, should an orbit be sustained, it could not be by a prime mover. There is no inconsistency."
      The Fist Way is not an argument for consistency.

      I can speculate that little angles nudge every particle in the universe along and they are all continuously battling little demons trying to slow them down, but the little angles typically win, and that is consistent with what we observe.

      The First Way is an attempt to prove the necessity of a first mover, and in that attempt the First Way fails.

      "It _is_ in an impeding medium."
      No, all motion is in space. You are in space. The medium does not impede motion, rather, neighbors within the medium impede motion of one object by gaining motion for themselves. No net motion is lost.

      "Absent the impeding medium which removes energy from objects in motion, a continuing force would cause the speed to go to infinity. According to special relativity,"
      According to special relativity the mass of an object increases as the speed of the accelerated object approaches c. Energy is not lost, it is gained by the object being accelerated , or gained by any other object the accelerated object loses energy to. E=mc^2 is an expression of the conservation of material.
      (cont.)

      Delete
    12. Tom (cont.),
      "As for "Carrasquillo", I don't much care what he says."
      Feser provides a link to his work on the right side column of resources on this site, Ite ad Thomam. If you were to study his syllogistic interpretation of the First Way you would likely gain knowledge into the structure of the logical arguments Aquinas employed.

      "I am sorry, but you do not seem to understand what their results say."
      Aside from the defects I have previously demonstrated the First Way is also logically invalid as an argument for the necessity of the first mover in that it employs the logical fallacy of false dichotomy, namely that a thing must either move itself or inactively be moved by another. Aquinas crucially failed to account for the case where two objects, each incapable of moving themselves, have the ability to actively move each other. This mutual repulsion or mutual attraction is in fact how motion occurs at the most fundamental level. Macro examples abound, such as a rocket, which moves itself through space using the principle popularized as "for every action there is an equal and opposite reaction".

      Thus, a gross error of Aquinas, one he certainly had access to in his time and glaringly failed to realize, is in this statement "It is therefore impossible that in the same respect and in the same way a thing should be both mover and moved, i.e. that it should move itself."

      You move yourself even when you hold your breath. An underwater mammal is a clear example of an object that moves itself. Aquinas failed to realize this obvious fact.

      Other examples of objects that move themselves abound, such as the aforementioned rocket, also a submarine boat, timed reactive device, battery operated clock, self drive electric vehicle, the sun and every other star, and on and on, making this error of Aquinas rather egregious.

      At the base of these cases of objects that are in fact both mover and moved, that do in fact move themselves, is the mechanism of mutual motion of objects that otherwise cannot move themselves, say, two electrons, or an electron and a proton, each of which cannot move itself if alone in space, but can mutually move each other via the electrostatic force.

      Delete
    13. @ StardustyPsyche,

      "You may observe that an object comes to a rest, but what you are failing to realize is that other objects increase in their velocity by an amount equal to the observed loss."

      Actually, the Newtonian formulation is _not_ that velocity is conserved. It is that _momentum_ is conserved, _if_ it is measured in _inertial_ coordinates. Furthermore, Newton could not define inertial coordinates, as he well knew. Rest, with respect to the stars, which are in accelerated motion, doesn't work. Still, you ought to get straight the difference between momentum and velocity before you "explain" your prejudice to me.

      "No, all motion is in space. You are in space. The medium does not impede motion, rather, neighbors within the medium impede motion of one object by gaining motion for themselves. No net motion is lost"

      This is just garbage. If two blobs of putty collide and stick, motion is lost in any inertial reference frame.

      "According to special relativity the mass of an object increases as the speed of the accelerated object approaches c. Energy is not lost, it is gained by the object being accelerated , or gained by any other object the accelerated object loses energy to. E=mc^2 is an expression of the conservation of material."

      You cut off my argument, and substituted something irrelevant to what I said, I guess because you didn't understand it although I put it in pretty simple terms. If an object moves from one place to another with 0 time elapsed on its clock, it is moving at infinite speed. If it changes in a short distance it can't be moving at infinite speed. Populations of neutrinos, which interact so minutely with matter that they can travel tens of thousands of light years through lead without interacting were found to be changing between types in a distance as short as the Earth's diameter. This meant that they had rest mass, and that meant that they couldn't be "seeing" time intervals of 0 in their forward direction because if they could they would have to have infinite energy according to special relativity (don't interrupt me again). However, if a constant force were applied in the same direction, its velocity would indeed approach infinity because it would see the time interval over long distances approaching 0 and it would be 0 in the infinite limit. Another way of seeing this is that the space interval in the direction of motion would approach 0 for the forever accelerated object and this would make the distance crossed 0 and it would "cross the universe" in 0 time, although unaccelerated observers would see it as a great distance crossed in great time.

      So if Aristotle was worried that a mover operating forever would make the speed infinite, he was right even though his concept of motion did not come from modern science but from a different direction that did not contradict science at all.

      Since you confuse velocity, momentum, energy, relative motion and absolute motion and change over time with change in the present, and the very meaning of change, I don't think you are doing anything but bootstrapping radical prejudice which blinds you from seeing that your assumptions grossly mislead you and anyone else who takes you seriously.

      Also, I doubt that Professor Feser is anywhere as close in agreement with you as you seem to think.

      Tom Cohoe

      Delete
    14. Tom,
      In Newtonian mechanics both momentum and energy are conserved, I did not bother going into those details, as I left it to the reader to understand the relationships between velocity, mass, momentum, and kinetic energy.

      "This is just garbage. If two blobs of putty collide and stick, motion is lost in any inertial reference frame."
      No, and this is the core misconception in the A-T argument for the first mover.

      Aristotle, Aquinas, Feser, Tom, and most of the good posters here have this point wrong. This ancient error you all continue to hold is at the heart of your misunderstanding.

      The kinetic energy of motion of the two blobs of putty is converted into the kinetic energy of motion of the molecules that make up the blobs of putty, what we call heat.

      No motion is lost, in the very real sense that matter/energy is considered, that is, the molecular motion of the constituents of the putty increases by an amount equal to the decrease in linear translational motion the blobs of putty started with, in a manner that conserves material (matter/energy).

      More extreme examples are kinetic weapons and meteors from space. The motion of these projectiles is never lost, rather, it is transformed into the molecular motion we sense as heat.

      That is true of all objects that appear to the human senses to slow and stop. To our immediate senses the motion seems to be lost, and that apparent loss leads one to argue for a first mover.

      But, in truth, motion is never lost, only transformed. Thus, no first mover is called for.

      At base causality, to the extent that causality is a valid concept at all, is fundamentally circular, not hierarchical.

      We don't need a first mover at the base of a hierarchical regress from moved to mover because at base objects move each other in net lossless perpetual mutual interactions that are fundamentally circular, not hierarchical.

      Delete
    15. @ StardustyPsyche

      In Newtonian mechanics, energy is proportional to the square of momentum. Energy is a far more complicated notion then momentum and the conservation of energy as a principle came long after Newton. It must be constantly revised to account for new findings. Leibniz understood the conservation of kinetic and potential mechanical energy but that was only the beginning and he didn't call it energy. Meanwhile, Aristotle had said the equivalent of "Energy is a condition that describes the capacity to do work". That's not bad for someone the Aristotle denouncers always misunderstand and denounce as an illogical step on their illogical denunciation of Aquinas.

      There was no speed limit in Newtonian mechanics and Aristotle's supposed worry that a prime mover acting forever on an object could move it to a speed that in the infinite limit was infinite is in no way inconsistent with Newtonian mechanics.

      If you would understand what I've tried twice already using the discovery that neutrinos have mass, you would also understand that the speed of light as an absolute speed limit is also not inconsistent with this so-called worry of Aristotle's. Perhaps it would have been simpler to point out that in Einstein's relativity, space intervals in the direction of motion grow smaller proper to the object in relative motion and become 0 in the infinite limit so that an object with 0 rest mass covers any space interval in 0 proper time, corresponding to infinite speed in spacetime events separated by finite time and space in a frame in which objects with non zero rest mass (that's you and me) are at rest. The 0 space limit that an object acted upon by the prime mover forever would measure in its tangent bundle spacetime manifold (inertial reference frame in curved spacetime) is completely consistent with Aristotle's supposed "worry".

      I have pointed out that Aristotle did not forecast modern relativity, but that this supposed worry is completely CONSISTENT with it. This is the same as that understanding that a ball dropped from a building will fall to the ground is not the modern science of gravity but is completely CONSISTENT with it.

      What happens with an individual object is not what happens with a manifold of spacetime events in Einstein's relativity. The development in time of such a manifold is not predicted by relativity because position and momentum (or time and energy) are not precisely definable at any single spacetime event, so the whole manifold of spacetime events is not predicted, uncertainty growing in the time coordinate from any defined manifold of spacetime events at a particular time (events separated by 0 interval) on that manifold.

      This you do not seem to understand or want to understand.

      "Russell found that scarcely a sentence [that Aristotle and the ancient philosophers] wrote was correct."

      Ha-ha. Why if Russell said it, it has to be the truth, huh.

      "At base causality, to the extent that causality is a valid concept at all, is fundamentally circular, not hierarchical"

      That causality is of God and that the principles of science are a limited effect of God's timeless will does not seem to be a concept that you are willing to entertain, so the only thing that is circular here is your reasoning.

      :-)

      Tom Cohoe

      Delete
    16. Tom,

      Good luck trying to argue physics with someone who doesn't understand the definitions of terms.

      I know you understand, but perhaps some readers don't.

      Motion in physics is velocity. Velocity is a vector not a scalar so it is not just dx/dt but dx/dt with a direction. So when a thing changes direction or slows down motion/velocity is not conserved....by definition.

      Momentum and energy have different definitions. The First Mover argument is about local motion/velocity, not momentum/energy.

      Regarding objects causing mutual motion (in this case objects orbiting each other), here is what Ed had to say:

      https://edwardfeser.blogspot.com/2012/06/oerter-on-motion-and-first-mover.html

      The trouble with this is that it assumes that the Scholastic is arguing for a “first mover” in the sense of a mover coming at the head of a line. And as I have just explained, that is not what is at issue. Let a system of causes be as complex as you wish -- an infinite series, a circle, a vast crisscrossing network -- to the extent that they are instrumental in character, there will have to be something outside the system as a whole imparting causality to it.

      Delete
    17. @ bmiller,

      Speed is the magnitude of velocity. If one speaks of the velocity of something constrained to move in a one dimensional manifold, speed is the same thing. I was speaking of velocity in the direction of an applied force, so "speed" was adequate and simpler than "velocity".

      I have to add that where I mentioned "tangent bundle" I should have said "tangent space".

      Tom Cohoe

      Delete
    18. Hi Tom,

      I was referring to Stardusty's confusion wrt motion/velocity and energy and momentum conservation, not anything you said.

      I had an exchange with him years ago and I see that he still has the same misconceptions.

      Delete
    19. Tom,
      Material interactions are net lossless.

      When 2 blobs of putty splat their motion seems to be lost. That is a core misunderstanding of Aristotle that makes the argument from motion, the First Way, unsound.

      I left it to the reader to understand the generalized sense of the word "motion" in this context, but I see that was not to be. More specifically, the kinetic energy of the blobs of putty is not lost, only transformed. For example, one of the simplest transformations to realize is that the kinetic energy of motion of the blobs of putty, considered as whole objects, is transformed into the kinetic energy of molecular motion, what we call heat.

      The same holds true for any observable process that seems to require a continual input of energy to keep it going. What appears to be what we now know as frictional losses are not actually net losses at all. You can get a sense of this by simply rubbing your hands together, whereby the energy needed to move your hands against an apparent resistance is transformed into heat, which is the energy of molecular motion.

      Aristotle had the mistaken notion that objects would slow and stop and their motion would be lost, and this leads to an argument for a first mover. While it is true that the obvious movement of an object will stop absent an external mover to keep it going, what Aristotle failed to realize is that the energy of the motion of the observed object was in fact transformed into other forms of energy in a net lossless transformation.

      Because there is no net loss of material, that is matter/energy that are equivalent, there is no necessity for a first mover to sustain our world of observed motions.

      "...completely CONSISTENT with it."
      The First Way is not an argument for CONSISTENCEY, rather, the First Way is an argument for NECESSITY.

      One can concoct an unbounded number of unevidenced speculations that are CONSISTENT with modern physics, no prizes for such fanciful prattle.

      "Therefore it is necessary to arrive at a first mover" is what Aquinas claimed. That claim is unsound because the First Way employs multiple false premises as an argument for the NESESSITY of a first mover, and is logically invalid as an argument for the existence of god.

      Delete
    20. @ StardustyPsyche,

      "That is a core misunderstanding of Aristotle that makes the argument from motion, the First Way, unsound."

      Oh well, then I guess all those guys before the understanding of black body radiation, that besides convection and conduction loss of heat there was radiation that carried momentum and energy away from the blob of putty, Planck's law, and all that end of the 19th century stuff, had a core misunderstanding of momentum and energy and were unsound thinkers. We can keep the time at which unsound thinkers still held sway right up to your inability to understand that infinite speed would be the limiting speed in the direction of motion of a non zero rest mass object acted on forever by a force in the direction of motion.

      Voila, by your type of "reasoning", you yourself are an unsound thinker.

      :-)

      "there is no necessity for a first mover to sustain our world of observed motions."

      Because you are unable to entertain any other ideas, that is merely your assumption. When you put down your assumption as your conclusion, you are just going in circles.

      :-)

      "fanciful prattle"

      More evidence that you are prejudiced by a bias that has trapped you in circular reasoning - call what you can't explain away given your initial unwarranted assumption, "fanciful prattle".

      Ha-ha-ha! This is better than the weekend funny papers, watching you floundering desperately, stuck in putty.

      Humility is the way out.

      :-)

      Tom Cohoe

      Delete
    21. Tom,
      "Because you are unable to entertain any other ideas"
      I entertained the idea that maybe Aquinas had some kind of sound argument.

      Then I analyzed the First Way in detail, its premises, and its logical structure.

      I identified several false premises and multiple instances of invalid logic in the First Way.

      Recall, the First Way is "The first and more manifest way", in other words, Aquinas led with his best, his strongest, argument.

      If the First Way is unsound then the foundation of A-T is unsound.

      The First Way is unsound.

      The human body, which is used in an example of objects not being able to move themselves, does in fact move itself, so Aquinas presents a false premise there.

      An object in motion does not move "only" because it is being moved by another, since an object in inertial motion is not being moved by another, so there is another false premise by Aquinas.

      When Aquinas says "because then there would be no first mover" he is begging the question by using the conclusion of the argument as a premise in the argument.

      The First Way is founded on the false premise that absent an external mover a moving object will slow and stop and its motion will be lost and disappear from the universe.

      Aquinas employs a false dichotomy between an infinite hierarchical regress or a finite hierarchical regress terminating in a single first mover, neglecting the third alternative of a circular regress terminating finitely in a multitude of mutual movers.

      The last clause "; and this everyone understands to be God." is logically invalid in that there is no logical connection with the prior argument.

      Koons has identified the lack of connection between god and the rest of the argument as the "gap" problem, in other words, the non-sequitur logical fallacy.

      The argument from motion is based on what is "evident to our senses, that in the world some things are in motion", thus, physics. Koons tries to make some rather weak points that A-T is compatible with QM. Bmiller above kindly provided a link to an old Feser post wherein Feser argues for compatibility between inertial motion and a first mover.

      Both Koons and Feser make the same error you made, in that compatibility is irrelevant. The First Way is not an argument for compatibility, it is an argument for necessity.

      Aquinas falsely claims "Therefore it is necessary to arrive at a first mover" when in fact his argument is riddled with defects and is not worth the paper it was written on.



      Delete
    22. @ StardustyPsyche,

      "The human body, which is used in an example of objects not being able to move themselves, does in fact move itself, so Aquinas presents a false premise there."

      Aquinas means that non-material mind is involved in the human body's motion. That this contradicts a false assumption of yours does not make it false.

      "An object in motion does not move "only" because it is being moved by another, since an object in inertial motion is not being moved by another, so there is another false premise by Aquinas."

      You seem to think that there is nothing but inertial motion. One can predict nothing with inertial motion because there is growing uncertainty in motion. You can't entertain the consequences of this because it violates where you want to go.

      "The First Way is founded on the false premise that absent an external mover a moving object will slow and stop and its motion will be lost and disappear from the universe."

      As I already pointed out, the medium is always resistive. Appeal to microscopic motion and on until Planck, if your type of argument is valid, makes all physicists not worth the paper they wrote on because they did not and still do not understand what is going on. Neither do you. Neither you nor anyone else understands what is going on. Fitting your conclusions to your biased assumptions is just, as you say, "begging the question"

      What do you think you are doing anyway?

      "The First Way is not an argument for compatibility, it is an argument for necessity"

      Funny! It cannot be necessary if it is compatible! Do you really think this way?

      Tom Cohoe

      Delete
    23. Chapter 13:3 of Summa Contra Gentiles presents the First Way and the reasons supporting it including all distinctions.

      I think it's a better place to start the the Summa Theologica. The argument is succinct and the support is all in one place.

      Delete
    24. @ StardustyPsyche,

      "What does it mean to exist? [etc.]"

      You may worry about such matters but why should we feel you deserve answers to your avowedly purposeless postings (even though you pose as a teacher)?

      "I would say that material [...]"

      Everyone has a right to an opinion, even if having it is purposeless or emotional projection, but that right doesn't make your opinion worth beans.

      "Material exists and form is a property of material that has no independent existence."

      I take this to be only another part of your emotional projection. But good for you for having opinions.

      "Any notion of purpose for the rock is a matter of human projection."

      Your talk about it is your own projection … and that is just your own speculation.

      "The Fifth way is arguably the worst in a list of arguments by Aquinas that start out bad at First and only get worse from there."

      Unfortunately, your argument is full of lacunae in which you blithely skip over things you cannot understand because your purpose (which you really do have) entails not understanding them.

      Tom Cohoe

      Delete
    25. Tom,
      "As I already pointed out, the medium is always resistive."
      That is the prime fundamental error of Aristotle, Aquinas, Feser, and you.

      The medium we are in is space. Space, for material motion, is net lossless. When material moves through space, what Thomists call local motion, change of place, no net material is lost, only transferred or transformed.

      That fact makes a first mover unnecessary by what is evident to the senses.

      "Funny! It cannot be necessary if it is compatible! Do you really think this way?"
      No, those are your words, not mine.

      The First Way is an argument for the NECESSITY of a first mover. Aquinas attempted to use what is evident to the senses to argue for the NECESSITY of a first mover.

      Aquinas used multiple false premises and invalid logic, therefore his argument is unsound and he failed to prove the NECESSITY of a first mover, and failed to prove the existence of god.

      Delete
    26. @Stardusty,

      Some years ago Dennis Bonnette on Strange Notions put up a post that argued to a first mover from a body's change of location. If I capture Bonnette's meaning right, he argued so:
      body B in linear locomotion at velocity V exists/is at location L in relation to bodies C and D at time T, and B exists/is at V at L1 in relation to C and D at T1.
      Therefore B existed at L1 potentially at T but exists at L1 actually at T1.
      Therefore B underwent a transition from potentiality to actuality under the category of location although not under the category that comprises its velocity.
      But transition from potentiality to actuality under ANY category is motion.
      But a substance existing potentially under some category does not have the power to give itself actual existence/being under the given category, by the principle of proportionate causality - i.e. something cannot give what it does not have. In this example, B doesn't have existence at L1 at T, so it can't be the cause of its own existence at L1 at T1.
      Therefore something else, which has the power to confer location at L1, must bring B from potential existence at L1 to actual existence at L1 - since B can't effect that change/motion itself.

      Therefore even cases of change of location by a body moving at constant velocity are cases of "motion," and require a prior mover. Therefore attacks on Aristotelian/Thomistic explanations of locomotion fail to undermine the First Way. ANY change requires a changer that is already actual (or that is so virtually; God is not undergoing locomotion but has the power/virtue to effect locomotion).

      IOW change of position is a "new existence;" B is not as it was before.

      https://strangenotions.com/how-new-existence-implies-god/

      Do you have views about this argument?


      Delete
    27. @ StardustyPsyche,

      In Summa Theologiae, Part 1, Question 2, Aquinas ends his "first way" with the words "and this everyone understands to be God." He argues that everything is moved by something else. That is true, Gravity and other forces cause the motion of things. Things in motion radiate their energy, either to infinity or to other objects, so you can say that space is a resistive medium. You take a position that amounts to "nothing is lost" which is, effectively, that "everything is one thing", an absurdity. Two apples forced apart by a hand that takes them from a basket and throws one away will return to the other apple by gravity, even though a light beam initiated on the same trajectory as the tossed apple will not return. The medium, through gravity, resists the motion of the apple and its energy, measured from the point of view of the apple in the basket, is radiated into space which is how space is a resistive medium. Of course, the apple is likely to collide with something else before it can return to the basket, in which case energy will be radiated into space from the collision, as extensively discussed previously.

      You may not wish space to be a resistive medium, but that does not make it so.

      You claimed that Aristotle thought that a force applied to an object forever would cause it to move infinitely fast, but this is correct. You have repeatedly ignored this consequence of relativity, so your claim about Aristotle was false.

      I cannot take your "teaching" seriously at all. I still want to know what your purpose is in what is clearly just another boring attack on the Catholic Church.

      Tom Cohoe

      Delete
    28. ficino4ml,

      I haven't read the link to Bonnette yet, but I think that what you wrote captures a lot of what Aquinas argued for wrt the First Way in Summa Contra Gentiles. It seems that Bonnette goes further than the SCG and actually presents the Aristotelian argument with all the details.

      I posted a link to the relevant section above and there Aquinas references the sections from Aristotle's Physics that apply.

      I found that it's easier to read Aquinas' commentary on it than Aristotle himself. It's been quite a while since I read it for myself though

      Delete
    29. ficino,
      Dr. Bonnette is declaring principles ad hoc, conflating existence with motion of an existent object, and then circularly employing his ad hoc assumptions.

      Even Aristotle understood that motion would continue ad infinitum absent an impeding medium, and did not himself apply an act/pot analysis in the case of such unimpeded motion. This is a fact of history the A-T folks seem to be avoiding, either intentionally or out of mere lack of study.

      Let's see what Aristotle had to say on this subject:
      archive.org/stream/aristotle-physics-book-III-and-IV-clarendon/aristotle-physics-book-III-and-IV-clarendon_djvu.txt
      CLARENDON ARISTOTLE SERIES
      General Editors
      J. L. ACKRILL and LINDSAY JUDSON
      IV. 8 TRANSLATION 214 b
      (4) Again, no one could say why something moved will come to rest somewhere; why should it do so here rather than there? Hence it will either remain at rest or must move
      on to infinity unless something stronger hinders it.

      So here we have Aristotle describing, essentially, inertial motion. Note, Aristotle makes no attempt to analyze such motion using potential versus actual positions, or anything of the sort, rather just the opposite.

      Aristotle understood that once an object is moved it will move on to infinity unless something else hinders it.

      Aristotle understood that motion from L to L1 is not the actualization of a potential that requires an external mover, rather, motion of a thing is a natural state of that thing, once moved, and that thing will remain in that state of motion unless "something stronger hinders it".

      The error Aristotle made is in believing that sub-lunary motion is in an impeding medium such that the motion of a thing (or what we would call the kinetic energy, or a different parameter we would call the momentum), is lost. It is the perception of loss that was the key mistake.

      Dr. Bonnette lacks the insight of Aristotle in analyzing motion absent a lossy impeding medium and has invented ad hoc a baseless and false principle that somehow requires a mover to keep it moving. Obviously, the opposite the case. It is the stopping of a moving object that requires an external mover, or as Aristotle correctly stated "something stronger hinders it".

      Both Dr. Bonnette and Dr. Feser also, and similarly, have existential inertia completely backwards. An object in existence at T and then in existence at T1 has not changed in its aspect of existence, therefore, no changer is called for to account for existential inertia. Yet, Feser has repeatedly asserted just the opposite, that somehow an object would just blink out of existence absent an unseen changer, and therefore there is a first sustainer somehow keeping everything in existence, and this is related to the very confused argument put forth by Dr. Bonnette with respect to existence at different times and places. Blinking out of existence would be a change calling for a changer, continued existence is not a change and therefore does not call for a changer.

      Delete
    30. Tom,
      "is radiated into space which is how space is a resistive medium"
      Ordinary object do radiate energy, true, but that does not make space a medium that is resistive to motion, nor does the radiation of energy by objects mean there is a net loss of energy, there isn't. Matter/energy are conserved.

      Air, for example, seems to be a resistive medium, commonly called air resistance. An object in motion does indeed lose energy to the air as it moves through the air, and from the direct human senses that energy appears to be lost.

      Such apparent losses are all around us. Roll a ball and of course, it will slow and stop and its energy of motion appears to have been lost. But that appearance turns out to be wrong. In fact the energy of motion of the object was merely transferred to increased energy of motion of the molecules that air is composed of.

      "You take a position that amounts to "nothing is lost" which is, effectively, that "everything is one thing", an absurdity."
      Everything is part of just one system, the cosmos as a whole. There is no necessity for a first mover to account for the observed motions within the cosmos because the causes of these myriad motions are the mutual and net lossless interactions of everything already moving in the cosmos.

      "You claimed that Aristotle thought that a force applied to an object forever would cause it to move infinitely fast, but this is correct. You have repeatedly ignored this consequence of relativity,"
      According to relativity c, the speed of light in a vacuum, is an absolute speed limit. Thus far, all experiments performed confirm this.

      Acceleration is a process of applying a force over time. In particle accelerators force is applied to particles over time to accelerate them. It has been confirmed experimentally that the more a particle is accelerated the more its speed asymptotically approaches c, but the energy of that acceleration is not lost, rather, the mass of the particle increases.

      So, I suggest you do a little more research on the subject of accelerating beyond c, and you will find you need to modify your views on this subject with respect to the predictions of relativity you state above.

      "I still want to know what your purpose is in what is clearly just another boring attack on the Catholic Church."
      I am not interested in assertions of personal motives. I am interested in logical arguments and independently verifiable facts. Assertions of personal motives are unverifiable, but you asked, so I will tell you for whatever it might be worth to you, perhaps nothing at all. I am an American and we Americans typically tend to regard competition as a sound means to develop the best solutions. I find engagement with agreeable folks to be both boring and likely to suffer from mutual confirmation bias, sometimes called an echo chamber. I prefer to test my solutions in a marketplace of competing ideas.


      Delete
    31. @ StardustyPsyche,

      "According to relativity c, the speed of light in a vacuum, is an absolute speed limit. Thus far, all experiments performed confirm this."

      Take two cities in the US. People at rest in those cities and at rest between them measure their spatial separation as 1000 miles. A clock on a photon measures the time to travel between the two cities as 0. This is infinite speed even though the people at rest measure the speed of the photon as c. The photon measures 0 elapsed time wherever it goes.

      Your particle accelerator example considers only the reference frame of the experimenters. In fact, we measure the lifetime of unstable particles approaching the speed of light in an accelerator to be longer than the same particles at rest. This is the same effect for unstable particles with rest mass.

      "The error Aristotle made is in believing that sub-lunary motion is in an impeding medium such that the motion of a thing (or what we would call the kinetic energy, or a different parameter we would call the momentum), is lost. It is the perception of loss that was the key mistake."

      Only if there are not things would this be a mistake. The error that you make is beyond circularity. It is a tight little knot of thought collapsed into itself due to self-willed prejudice.

      I too am an American and I notice that you exclude anything from your mind that contradicts your prejudice, making your cogitations useless. The "marketplace of competing ideas" indeed! What you allow into your head anyway. Funny!

      🤣

      "sometimes called an echo chamber"

      I wouldn't project the biased interior of your skull onto the Catholic Church.

      😀

      Tom Cohoe

      Delete
    32. You are conflating acceleration of a massive particle with a quanta of electromagnetic energy. You and Aristotle were both wrong, accelerating a massive object continuously does not cause its speed to approach infinity, it causes the massive particle to asymptotically approach c, while gaining mass.

      You are also making the same mistake of considering an apparent medium as fundamental. The fundamental medium is space. You are in space, everything is in space, all the molecules of an apparent medium such as air or water, are in space.

      Your key error is not realizing that all interactions in space are net lossless, and only transfer energy/mass/velocity/momentum, or in a loose manner of speaking, motion.

      Objects may radiate in space but that does not make space an impeding medium. Space is a lossless medium. material/matter/energy are conserved in all interactions in space.

      It is the conservation of material in space that renders the First Way unsound, among other defects of Aquinas, Aristotle, Feser, you, and most of the good posters here.

      There is no NECESSITY for a first mover, as A-T falsely claims, because all interactions in space are net lossless.

      At base, everything in the universe moves each other, negating the NECESSITY for a single first mover.

      Delete
  25. Some people here keep repeating things like that particles have the same properties, such as mass, whether or not they are bound.

    They do not have the same mass. A particle dropped into an potential well emits energy as radiation as it falls into the well and that appears as a loss of mass, i.e. an electron bound within a human body has a lower mass than a (relatively) free electron outside the body.

    The mass of a free electron is:

    9.1093837015 × 10−31 kg (https://www.google.com/search?ie=UTF-8&client=ms-android-verizon&source=android-browser&q=mass+of+a+free+electron)

    Of a free proton is:

    1.67262 × 10−27 kg (https://www.google.com/search?ie=UTF-8&client=ms-android-verizon&source=android-browser&q=mass+of+a+free+proton)

    Of a free hydrogen atom is:

    1.67 × 10-27

    ReplyDelete
    Replies
    1. @ Anonymous,

      "Some people here keep repeating things like that particles have the same properties, such as mass, whether or not they are bound."

      This comment was by me. I accidently posted it before I was finished writing it. I decided to rewrite it in a simpler form. The rewritten version appears above.

      Tom Cohoe

      Delete
  26. David Mermin wrote a great article in Physics Today back in 2009 and said this about quantum fields: "...When I was a graduate student learning quantum field theory, I had a friend who was enchanted by the revelation that quantum fields were the real stuff that makes up the world. He reified quantum fields. But I hope you will agree that you are not a continuous field of operators on an infinite-dimensional Hilbert space. Nor, for that matter, is the page you are reading or the chair you are sitting in. Quantum fields are useful mathematical tools. They enable us to calculate things..." https://physicstoday.scitation.org/doi/10.1063/1.3141952

    ReplyDelete
    Replies
    1. "When asked … [about] an underlying quantum world, Bohr would answer, “There is no quantum world. There is only an abstract quantum physical description. It is wrong to think that the task of physics is to find out how nature is. Physics concerns what we can say about Nature.” — Niels Bohr

      Delete
  27. Kudos to the thinking of Mr.Digregorio and his diligence in parsing out what other thinkers and doers have thought and done. It reinforces my contention that physics is not philosophy, nor do their promises and distinctions rely upon one another. Nor did philosophy enable physics or the other way 'round. Which is, in any sense you wish to invent, or discount, cause for the separate realities of both. Ergo, ancients, who fumbled around with notions they did not understand, were not fully what they wanted to be.

    To repeat myself, they just tried harder to think better, and so on...

    Finally, I care little for the interests, preferences and motives driving most of this. None of them are on my agenda. No horse there. No dog, either.

    ReplyDelete
    Replies
    1. Paul D. Van Pelt,

      If "Nor did philosophy enable physics or the other way 'round.", then why did modern physics develop in Western Christendom and no where else?

      Delete
    2. The west developed the first mechanical clocks which is a prerequisite for modern physics.

      Delete
    3. From Wikipedia:

      In medieval Europe, purely mechanical clocks were developed after the invention of the bell-striking alarm, used to signal the correct time to ring monastic bells.

      Delete
    4. Why other civilizations did not create modern science HERE

      Delete
  28. Thanks for the remarks from Bohr. I had forgotten about him.

    ReplyDelete
  29. @Cantus
    @Tadeo
    @Talmid
    @ficino4ml

    Guys i just received today notifications for your messages ! I will read them all !! Thank you so much for your attention

    ReplyDelete
  30. @Tadeo

    I started reading the Last Superstition in portuguese, and youre right, It was intelligible untill they started talking about that universals subject. I just could'nt really understand that, and untill today that subject is confuse in my mind. Something about Ockhams nominals i guess. I understood that an important step is to understand deeply this universals idea, and i guess transcendency will seem more logical to me.

    ReplyDelete
    Replies
    1. @Rafa

      Hey bro, it's nice to see that our replies got to you! Sorry for taking so long to answer too - I ain't feeling very well.

      Going straight to the point I must say that the existence of universals is one of the most important points in philosophy because its consequences/ what follows from it have a huge impact on how the world really is.

      Nominalism has many varieties but I think it's safe to say that at the end of the day, all of them affirm the same damn thing (i.e the inexistence of universals) and fail exactly in it in different ways i.e they always smuggle universals in the background. It doesn't matter if its an Ockham variation, a Sellar's "metalinguistic" one, a buddhist one, or whatever (of course, to see how it fails specifically, you will have to dig into it, but there are other ways to see why this idea does not work in any way).

      I personally like Ed's work (I am a fan and I'm not gonna lie about it) and in his TLS book, he gives direct proofs for realism and indirect ones. Personally, I love indirect proofs - because I think that if you could show that your philosophical adversary can't get rid of the concept/ or even presuppose the very concept he's trying to deny it's pretty much a good reason for knowing that it sure does exist and its a real thing. That's exactly the case of nominalism i.e it needs universals to even get off the ground!!

      For example, when we (realists about universals) see a red thing and another one, it's pretty damn obvious that they share the exact same thing i.e redness even though they are different in other aspects. The nominalist will, of course, deny this fact. He's gonna say that "it's not the same redness" and that universals are "free creations of the human mind to facilitate speaking". But the basic problem that affects every single version of this idea is the fact the communication itself (i.e the proposition, the speech, the way he talks to other people) presupposes the very universals he's trying to deny and the fact that the conventional use or creation of things itself presuppose objective realities (just like to know what's an illusion or falsity itself presuppose a grasp of what truth is - it's a parasitic thing).

      So, we are human beings and WE DO entertain concepts. Of course, if Agnetha Falstkog is prettier than Olivia Newton-John or if Brazilian BBQ is better than feijoada is a matter of opinion, it's a subjective matter. But the point is, we do entertain universal concepts that not just must be but ARE the very same concepts that you and I must entertain and they are, by their very nature, objective. One simple example is mathematics. When I think about the Pythagorean theorem or the number 7 and you think the very same thing it's simply impossible for them to be different things i.e if someone understands the Pythagorean theorem at all - or Bhaskara or whatever - it is the same thing that we are talking about. It's not like you are entertaining your private concept of the Pythagorean theorem and I am entertaining mine because if it was really that way communication would be impossible.

      Consider science, for example. It is in the business of discovering the world around us - at least it should if not impregnated with ideology. But if scientists were not talking about atoms, electrons, spins, or whatever in a universal graspable manner i.e IN THE VERY SAME MANNER then we could only have particular concepts (that one shared by this scientist and another one shared by that other scientist) and we could not even be talking about the same thing - or discovering - the same thing. In short, realism is the only theory that does not turn scientific founding into a miracle - to paraphrase Hillary Putnam.

      Of course, Ed goes through reasons why we should reject this by a lot of different arguments, but I tried to sum up everything so you could understand better. There are a lot of other examples and vicious regress lurking nominalism and other arguments showing how they presuppose universals instead of getting rid of the body.
      (I will continue in just a sec)

      Delete
    2. (continuing...)

      But, if you are somewhat short-tempered like me (sometimes) - or allergic to bullshit - you could always ask a nominalist if he ever goes out of his grandma's basement to buy two packs of cigarettes (because if he says that he's not buying the same brand even though it's blatantly obvious in the very packages) or even see e-girls in the center of the city (because it's like seeing the same damn girl with tinted purple hair, sad eyes and bad taste for music everywhere) you could have a pretty good suspicion of his mental sanity. That's because even though my examples (cigarette brands or e-girl *culture*) are conventional things in the sense of being "creations of the mind" the very fact that they are sort-of conventions presupposes things that are themselves NOT conventions or creations of the mind i.e human beings and cigarettes and these last ones must be universals - at least in a sense - if they are to be instantiated at all.

      Hope that helps - and I hope you also chuckle a little bit at the example. Que Deus te abençoe sempre, cara!

      Delete
    3. Damn ! That really explains it all ! I will follow your advice and re-read it In English and I Hope I can understand it better than in Portuguese? Tell me something, how long have you been reading about these classical matters? What other social media do you use which we can chat better? I'm gonna give you my contact.

      rafaelnobre91@gmail.com

      I started reading that contra os acadêmicos list, I don't know if you know their work : Prolegomenos aos estudos o nome da lista, there they have an awesome list of books for those interested in follow an intellectual life, names such as, sertillanges, jean guitton, riboulet, Louis Lavelle, Ortega y Gasset, Erich Voegelin and many others. I would like to have some recommendations from you about how I should organize my studies..

      Delete
    4. Dear @Rafa

      It's been three years since I first met Ed's work and blog (in October of 2023, it will be 4!). I found his work (by chance) in the first place, but I must say that I am very, very grateful for this man's work. And it's not an overstatement to say that he saved and changed my life. I admire him so much - and I can affirm to you without hesitation that you will find his work very helpful throughout your life and philosophical inquiries.

      Before going into recommending books and authors I must warn you to not get philosophical ideas over your head. Do not overthink them too much. That's friendly advice. I have OCD and I must say that's a nightmare. Just to give you a comparison, back when I was 15 to 16 years old and I learned more about nazism, and socialism (e.g the great purge, the massacres against the Jews; started reading about Solzhenitsyn and Alexander Bezmenov, for example) I thought that nothing could be worse than all of that but - for my unfortunate surprise - there are philosophical ideas that can be very harmful too (for example, eliminativism, reductionism, anti-realism about persons) and they are simply difficult, at least for me, to get it out of my head - not because they are soo much convincing but because they are bizarre with even bizarre consequences and if you got to them off-guard and have OCD or the like, you will suffer with this. So, please, be very careful about it (even if you do not have any kind of psychological disease).

      There are a lot of things that I learned through all these years but you must note that I am NOT, in any important sense, an authority or expert in the matter (and my OCD blurs a lot of what I learned). First, to understand all of this right, I really think that Ed's work will be the best grounding for all of it. I do recommend you to read his book Aquinas (which I forgot to mention, but it's simply a very good book for beginners too!). You should also read his post's regarding Aristotelian-Thomistic Metaphysics to grasp everything better.

      But there are, of course, works and philosophers that I do recommend you read - starting with the *easiest ones*: Mortimer J. Adler (Aristotle's for everyone, for example, and the collab book Thomistic Psychology ); William Norris Clarke (The one and the many); Richard Percival Philips (Modern Thomistic manual); Reginald Garigou-Lagrange (Reality: a synthesis); Desiré-Joseph Mercier (Manual of modern scholastic philosophy vol.1); Steve J. Jensen (The human person. A beginner's Thomistic psychology); and last - but not least - the amazing book written by Tomás Alvira, Luís Clavell, and Tomás Mellendo Granado (Metaphysics, but I do recommend you to read the English version of it for the very same reasons you should read the TLS in English).

      The ones listed above are the *easiest* - or at least the most noob-friendly IMHO (that may be controversial and I am aware of that). There are, of course, other books that treat the matters more deeply but they are way, way harder to understand and I urge you to not dive into them - at least for now. Some examples are David Oderberg's (Real Essentialism) and Edward Jonathan Lowe (the possibility of metaphysics, for example). If you ever try to dive into them, I do recommend, of course, starting with Ed's work (especially Scholastic's Meta and Aristotle's Revenge - even though it's a book with difficult matters Ed makes a lot of it intelligible for laypeople) and Michael J. Loux (Metaphysics: a contemporary introduction) because they are very good books for beginners.

      (I will continue)

      Delete
    5. (Continuing...)

      At this point, you must be asking "why all that focus on Ed's work? are you just some kind of bizarre worshiper (or, like we say here in Brazil, a 'puxa-saco')?" And the answer is no! I just say a lot of his work in general because he is the kind of person that says what needs to be said. He goes straight to the point, he's incisive (in the good sense, of course) about his ideas. He will say firmly why an idea does not follow at all and make it clear about it (he will say the reasons why it does not follow and so on). In short, it's the kind of work that you will not find gaps in (and if you read pop philosophers and even other professional philosophers, you can clearly see how Ed is careful in his statements and analysis and do not fall prey to fallacies or misrepresent other positions).

      So, said all that, I think you now have a good arsenal of work on your side.

      Regarding social media, I don't have any besides Whats app, e-mail (I will e-mail you very soon), and blogger account (I only made one so people could see that's really me and not just some random, salty troll with hatred in his heart). My last social media account was Facebook (I never had a Twitter or Instagram or any of that) and I shut it down in 2016 because I made a post regarding Trump's getting elected. I stated that people should stop arguing about it and respect the American people's decision because it was their country, their decision and it was not of our business to criticize them. Sometime later my post had unexpected attention and a generalized discussion in the comments with ad hominem and everything you could imagine (and my post was just literally something inoffensive and simple as that! Without any direct or indirect offense) so I definitely decided to stay away from all of that cringe situation and shut it down (I don't miss it at all).

      Oh, and by the way, you may throw rocks at me by this (a lot of people do) but I'm not a fan of Olavo de Carvalho at all nor even the majority of the 'representatives' of the right-wing in Brazil (e.g Eneas, Plínio Salgado, etc.), but that's not because I'm leftist - not at all. But because I think that there are people that could be better qualified for making a case for conservatism, specifically. Some examples that I have in mind are John Kekes, Roger Scruton, and - in a sense my favorite because I like his sense of humor - David Stove. I just thought that it may be useful for you to know these authors too.

      Well, I think that's all, for now!

      Delete
    6. Whats'up Tadeo, hope youre doing good. I wrote a long text a few days ago, im not sure if it was posted here or there was some problem with it. Anyway, If it doesnt get posted i will take some time to write it again. In the mean time i just want to thank you for taking some time to answer me and helping me in this hard path. Im organizing my day in a way that i can dedicate daily time to read. My family had a serious problem and lately we are dealing constantly with the idea of tragedy and death, this kind of situation is helping me deal with the brevity of life and to evaluate what really means in life, what should I do with my time, knowing that death could be so near and lurking me.

      Delete
  31. As an old gerund grinder, I have no clue about Newtonian physics (I peeled rubber getting a B in college), let alone quantum mechanics. On this thread, I see Thomist A telling Skeptic B, you simply fail to understand P and Q. And then Skeptic B replies, no, you simply fail to understand P and Q.

    This is where disputes on here tend to end up. "You simply fail to understand Aquinas..."

    Meanwhile, in real life, millions of petitionary prayers go unanswered -- unless you're fine with the answer being, No. "Help me escape this predator." Nope, no escape, because the no escape was the gracious will of the First Unmoved Mover. No moving any agent to stop the predation because that just how the First Unmoved Mover rolls.

    What story will shut me up? "You simply fail to understand Aquinas"? "You just want to sin"? Something else?

    I have learned much from Prof. Feser's works and from this blog. The place where many discussions end is not a place of certitude, despite what is often claimed.


    ReplyDelete
    Replies
    1. One of the more sensible comments logged on this post. I could not have framed it better.

      Delete
    2. ficino4ml,

      The place where many discussions end is not a place of certitude, despite what is often claimed.

      What kind of "certitude" are you looking for? I doubt anyone here thinks they know all the answers. They are arguing for what they think are the best answers.

      Delete
    3. @bmiller

      There are different kinds of certitude? Tell me what are the kinds.

      We're often told by Thomists, and of course by the magisterium of the Catholic Church, that the truth of various propositions about God can be known with certitude by reason. Over on Strange Notions, Prof. Dennis Bonnette has argued many times that metaphysical first principles are known to be true with certitude. The point of this thread is not to debate physics for its own sake but to push back to metaphysical first principles, no?

      So for example -- realism about universals vs. nominalism. I don't get the sense that the realists on here generally think they don't have certitude about the truth of realism and the falsity of nominalism.

      Delete
    4. This comment has been removed by the author.

      Delete
    5. ficino4ml,

      Of course the Catholic Church declares that there are certain things that can be known with certitude but I as I recall those things are by revelation, not necessarily by reason. Reason only gets one so far, but it cannot conflict with revelation.

      Now of course if you don't believe in revelation you won't believe what Catholicism has to say anyway.

      The point of this thread, it seems to me from Thomists and others here, is to push back on the idea that physics is incompatible with God or in particular Christianity. Fair enough. You've been seeing both sides making their arguments. Do you think the "other side" lacks conviction?

      Regarding realism about universals vs. nominalism I think those posts were off-topic for this thread, but again, people are arguing for their positions.

      I think content of OPs and the quality of the commentators (generally) of Ed's blog are superior to most any other place for people interested in philosophy but maybe not well versed....like me.

      BTW, I am interested in the nuts and bolts of the physics arguments and many of the disagreements you've seen do not touch on metaphysical first principles at all, as Gaëtan Cantale-Miège has been pointing out.

      Cheers.

      Delete
    6. @bmiller: you wrote, "Of course the Catholic Church declares that there are certain things that can be known with certitude but I as I recall those things are by revelation, not necessarily by reason."

      ??? I just quoted Vatican I saying that God can be known with certainty ... "by the natural light of human reason."

      Perhaps you wrote your last before my last appeared in print here on the blog.

      Delete
    7. @bmiller: I posted an earlier reply but ignore it since it refers to yet another reply which I later deleted as too smart-mouthy.

      you wrote: "Of course the Catholic Church declares that there are certain things that can be known with certitude but I as I recall those things are by revelation, not necessarily by reason."

      I had quoted Vatican I saying that "If anyone says that the one, true God, our creator and lord, cannot be known with certainty from the things that have been made, by the natural light of human reason: let him be anathema." So certitude/certainty is claimed for knowing by natural reason the truth of metaphysical propositions like "God exists."

      You also wrote: "Do you think the "other side" lacks conviction?"

      As I understand the terms, conviction is a psychological property. It describes someone's attitude toward his/her beliefs. Certitude or certainty is an epistemic property; it applies to beliefs themselves, indicating that the person has no rational grounds for doubting the truth of the given belief. Or even put more strongly, it is factive, implying that the given belief actually counts as knowledge.

      Delete
    8. ficino4ml,

      Thanks for the quote from Vatican I.

      You're right that it's dogma that that the truth of various propositions about God can be known with certitude by reason. Especially "God exists". I was thinking of things that cannot be known by reason alone that are dogma also, like the Trinity. Of course I concur and you don't and so I suspect you're not a Catholic.

      People commenting here give reasons why they disagree and other people argue in response. The quality of argumentation here is generally better than elsewhere. So why do you think that it's not reasonable (as opposed to emotionally satisfying) to believe in God?

      Regarding certitude/conviction. I'm not clear what your point is. 2 sides are arguing for the position they believe in, I assume. Each think they possess knowledge. Is it an unfair advantage that people believe in Catholic teaching?

      Delete
    9. @bmiller:

      BTW we exchanged views years ago over on the classical theism forum, and I hadn't seen you "around" online until recently. I may have missed many of your posts since then. Anyway, hello again.

      I don't think I said that it's not reasonable to believe in God. I don't believe in God, and I don't find arguments for God's existence convincing. But certainly, people exercise their reason and come to the conclusion that God exists. I can't prove that their conclusion is false. I think the harder they drill down on the three "omni"s, the harder their case becomes to argue, but I'm aware of many arguments made.

      If I say that the truth of a proposition P is or can be known with certitude, I'm not talking about how strongly people (feel they) believe that P. I'm talking about a property of the proposition, such that there are no rational grounds on which P can be falsified. For example, we know with certitude that A = A, because that proposition is analytic. There are no rational grounds for proving it false. Whether you or I believe that A = A, or whether one of us believes it more strongly than the other, has to do with our individual intellectual biographies and not to do with the epistemic property of objective certitude that attaches to the belief and not to the believer.

      Does this make sense?

      Anyway, my original comment was prompted by my experience of observing and engaging in disputation about, not analytic propositions, but propositions that make existence claims about entities that are in principle not detectable by any methodology accepted by all parties in the disputation.


      Delete
    10. ficino4ml,

      Yes. Hello again. I do remember some exchanges. I can't remember the content, but then I can't remember what happened yesterday either ;-)

      I don't find arguments for God's existence convincing.

      As you might imagine, I don't find arguments for any alternative convincing. But I'm willing to listen.

      I'm talking about a property of the proposition, such that there are no rational grounds on which P can be falsified. For example, we know with certitude that A = A, because that proposition is analytic.

      I agree (although some people don't) but this is a tautology. Perhaps you mean syllogistic logic? But you and I can both believe in syllogistic logic and I believe I'm a brain in a jar while you believe that you really experience an external reality. How would you try to convince me otherwise? A real question BTW.

      but propositions that make existence claims about entities that are in principle not detectable by any methodology accepted by all parties in the disputation.

      Then what methodology in principle would you accept to prove that God exists? And conversely what methodology in principle do you think proves that God doesn't exist?

      Delete
    11. @bmiller: your questions call for what could be a long discussion, maybe potentially too long for a thread this old. And I am not the most learned discussion partner you'll find among skeptics.

      I suggested that "certitude" as an epistemic property signals a claim about a proposition, that there are no rational grounds for doubting its truth. If someone wants to deny axioms like the principle of identity or the PNC, I don't see how I can have a rational discussion with that person, since from A = ~A anything follows.

      I'd try to convince the solipsist to abandon solipsism (or convince the guy who believes that there is no external world to change his mind) by appealing to evidence that relies on evidence of the senses. I don't know that I can do more with someone who denies that the senses give us information about so-called external reality.

      I said earlier that I don't think I can prove that God doesn't exist. The most I can do is argue that attempts to conjoin certain claims about God (God as presented by Christianity and Judaism, let's say) tend to exploit slippage of meanings of words - e.g. "benevolent."

      So as not to make this reply too long, I'll just point to three reasons why decline to agree that conclusions of cosmological arguments and the like are known to be true with certitude:
      1. analogical predication introduces equivocation, although Thomists try to deny this; equivocation vitiates validity.
      2. consequences of the existential fallacy: it's invalid to derive an existence claim about particular from universal premises, and if the major premise is not universal, then AFAIK the argument won't be a deductive system. But certitude does not attach to conclusions of arguments that aren't deductive.
      3. we don't know that the major premises are true (e.g. if something begins to exist, then it has a cause).

      Delete
    12. ficino4ml,

      Regarding your 3 bullet points.

      It seems your point 1 is not a cosmological argument but is related to the attributes of God, right? I suppose if I didn't believe in God, then I certainly wouldn't believe in any arguments for His attributes. But your second point seems too broad for me to know which arguments you're referring to.

      Your 3rd is William Craig's favorite right? I don't see that argument defended here much. This is more the territory of the 5 Ways and Divine Simplicity.

      Regarding how to talk to a proported solipsist. I doubt I could talk him out of it since he must have considered all the arguments before. I suspect if he really lived his life as a true solipsist he would already be dead or locked up for his own safety. The fact that he could still function in society would make me doubt he is sincere. He relies on the reality that he denies.

      Delete
    13. ficino4ml,

      Sorry. Looking back, I don't detect a consistent "methodology" in your 3 points in response to my question:

      "Then what methodology in principle would you accept to prove that God exists? And conversely what methodology in principle do you think proves that God doesn't exist?"

      In fact, I don't detect a response to the second part of the question. "what methodology in principle do you think proves that God doesn't exist?"

      Delete
    14. ficino4ml,

      I'm an engineer by training. If I had unlimited resources, would have further studied physics and philosophy.

      I don't think gerund grinder describes the scope of your interests.

      If you don't mind, what is your training and interests?

      Delete
    15. @bmiller,

      I have a Ph.D. in Classics and publish on ancient philosophy and rhetoric. I really have no clue about physics!

      You are right, I have not proposed a methodology by which one might gain knowledge about God. It's not easy to know how to propose a methodology, since apologists for Christianity (I can't speak about apologists for Judaism or Islam or other religions) generally tell us both that God is ineffable and beyond our ken AND tell us many things about God.

      The best I can do is to suppose that experiences given to a wide range of people, across cultures and spaces, that can't reasonably be accounted for by natural causes, would provide evidence that God exists. After all, if God moves events in time-bound history (whether or not the "moving" itself was effected in time), people in general should be able to perceive them as moved by God and by no other mover, if they are to conclude from said events that God exists. Maybe, if the generality of people were given the same exact presentation at the same time with the same content, of which no natural entities could reasonably be considered the cause ...

      I guess I'm saying that to establish the existence of a singular entity, and not merely to argue that certain notions are mutually consistent, we need some event in time experienced by a wide swath of people, and verifiable. We have stories about such events ... e.g. Yahweh giving the covenant to all of Israel at Mt. Sinai, or the alleged events at Fatima ... but then, we are pushed back into evaluating historical sources.

      God is supposed to be the omnipotent and omniscient one around here, not me. If God wants to convince the generality of people that He exists, He should be able to figure out a way.

      My beef with analogical predication concerns even the Ways. St. Thomas insists that God is not a member of any genus. But for the First and Second Ways to go through, as far as I can see, the first unmoved mover and the first uncaused cause must be members of the genera, mover and cause. So not all that is true of the first mover (it's supreme in its genus) is true of God (in no genus). So the first mover is not God, because not all that is true of the one is true of the other.
      Similar problems beset other Ways.

      As for the premise, "if it has a beginning, then it has a cause," it doesn't matter whether St. Thomas endorsed the KCA. I gave that only as an example of how we can find arguments for God employing premises, which we do not know are true.

      Delete
    16. ficino4ml,

      I think most people throughout history have have been theists just by looking at creation. The rise in atheism is pretty new.
      I don't think it's because we've become smarter.

      Regarding St. Thomas. Too bad we can't ask him what he meant eh? But do you really think that there is an unmoved mover but it is not God merely because Aquinas put Him in the wrong genus?

      If God wants to convince the generality of people that He exists, He should be able to figure out a way.

      It seems that even witnessing miracles didn't convince some people. It seems that He allows free will and as long as that's allowed, people can choose not to believe.

      Delete
    17. @bmiller: I spoke too quickly when I said "for the First and Second Ways to go through...", as though they are sound arguments if God can be a member of a genus. I don't know that all their premises are true, since most of the premises reflect positions in Aristotelian ontology. I'll note though that Aquinas did not put God (your "Him", right?) in the wrong genus; he did not put God in any genus at all.

      Delete
    18. @ ficino4ml,

      I think that Aquinas was certainly smart enough to know that we cannot talk about God per se. We can only talk about a less than perfect image of God, but Aquinas, for the sake of clear and simple speach, doesn't want to make a muddying and mealy mouthed insertion of "image of", which adds nothing, every time he speaks of God. So he talks about God as "the unmoved mover", etc, even though God is not properly referred to as in the genus "mover".

      This is the way I see such things. We cannot speak above the level of our own intellect. Trying to just leads to a lot of confusing arm waving (some people seem to like that).

      😏

      Tom Cohoe

      Delete
    19. ficino4ml,

      I'll note though that Aquinas did not put God (your "Him", right?) in the wrong genus; he did not put God in any genus at all.

      Haha. OK.
      So if there is only a single unique thing, then is it proper to list it as a part of a genus?

      Delete
    20. @ Tom Cohoe: fine. Then we should not say that the conclusions of deductive systems that employ analogical predication are known with certitude to be true.

      @bmiller: no.

      Delete
    21. So he talks about God as "the unmoved mover", etc, even though God is not properly referred to as in the genus "mover".

      SCG-Chapter 13. Gives the various detailed arguments supporting the First Way.

      I think something that is usually missed is that A is talking about what moves things primarily. Anything that comes into existence cannot be the cause of it's own existence and therefore cannot be the primary cause of it's own motion. It's either moved directly by another, moved by nature (gravity for instance) or because of it's specific nature (in the case of animals, it's the generating cause that is primarily responsible).

      [8] In the second way, Aristotle proves the proposition by induction [ Physics VIII, 4]. Whatever is moved by accident is not moved by itself, since it is moved upon the motion of another. So, too, as is evident, what is moved by violence is not moved by itself. Nor are those beings moved by themselves that are moved by their nature as being moved from within; such is the case with animals, which evidently are moved by the soul. Nor, again, is this true of those beings, such as heavy and light bodies, which are moved through nature. For such beings are moved by the generating cause and the cause removing impediments. Now, whatever is moved is moved through itself or by accident. If it is moved through itself, then it is moved either violently or by nature; if by nature, then either through itself, as the animal, or not through itself, as heavy and light bodies. Therefore, everything that is moved is moved by another.

      If this is the case, there is only one primary mover.

      Delete
    22. @ ficino4ml,

      "Then we should not say that the conclusions of deductive systems that employ analogical predication are known with certitude to be true"

      In order to avoid the arm waving branch of discussion, I must first ask "known with certitude by whom?" for you answered me with "we". Unfortunately, a great argument according to one is humbug to another. One individual, or a group, does not add up to "we". So how are "we" to know anything with certitude. A big sign does not light up in the sky before all.

      "We" choose as individuals to believe this, or its essential opposite, that.

      Catholics (not all, of course) believe that enough sign is there to allow us to choose the essential truth without being so overwhelmed by some deductive system that faith is unnecessary ("faith is [...] the evidence of things not seen" - Hebrews 11:1 KJV).

      Aquinas had faith long before he wrote The Summa, so what he wrote must be seen in that light. He also wrote very early in the Summa that if we had nothing at all in common with those who oppose the faith, we could convince them of nothing, but knowing the truth ourselves, we could see how they were wrong because truth cannot oppose truth (in the sense of LNC).

      So his First Way is "natural theology" as philosophy, but it is written that it will not conflict with his faith.

      I was not overwhelmed by a deductive system at all. After 65 years of perplexity, I saw enough sign that I could choose (without "seeing"). I chose to believe.

      I do not have sufficient certitude that I think I could so overwhelm you with deductive logic that you would be forced to believe.

      😏

      Tom Cohoe

      Delete
    23. @bmiller: "If this is the case, there is only one primary mover."

      ??? We all know that Aquinas argues that there is only one primary mover.

      Delete
    24. @Tom Cohoe,

      Yes, different philosophers seem to have lists of different length when they enumerate what propositions can be known as true with certitude... and other philosophers may want to deny "certitude" most or all the time.

      My opining that touched off this subthread had to do with certitude about elements of what we might call "God talk." I'm not aware that there are many propositions beyond "God exists" that the Catholic Church teaches can be known to be true with certainty/certitude by natural reason. But "God exists" is one.

      Professor Feser wrote an article on it on Strange Notions some time ago:

      https://strangenotions.com/can-we-know-gods-existence-with-certainty/

      If you're interested in discussing "certitude" further, perhaps that could be a springboard. Since the present thread is almost 200 comments, I don't know how far further discussion will get on this thread.

      Delete
    25. @ ficino4ml,

      It is an annoyance but not that difficult to talk past the 200 comments limit. But again, "what can be known to be true with certainty/certitude" … by whom? By one, by many, by all? Can certainty be communicated through talk or does it require individual, private, study? It is one thing for Church documents to assert that something can be known with certainty and another thing for the way to this certain knowledge to be itself known.

      I am not even sure what "certainty" means. I speak of words and their definitions in words, not the state of my mind.

      Tom Cohoe

      Delete
  32. I would like to gently point out that space and time do not have inherent existence, but rather are epiphenomena or, more accurately, are literary devices within the framework of scientific discourse. Motion is only a displacement "in space and time" as a linguistic construct; motion is simply the change in state of any entity. If an entity does not undergo a change in state, nothing occurs and there is no "flowing time" for it. Time is only a numbering of different changes in state, just as space is only the enumeration of the extension of a body.
    Therefore, when Aristotle states that a mobile does not move as long as its state changes, he is correct. Without a cause, it does not change state and does not move. His physics states something fundamental with respect to the symmetries observed in physics, the first being that of Galileo's group, which states exactly this and that is that the laws of physics at rest or in constant motion (i.e., when the entity does not change state even if the observer is at rest or in motion with respect to it) are the same; not only that, but this is also consistent with the Lorentz group, and that is, if an entity is constantly accelerated in a gravitational field and, therefore does not change state while always remaining equally accelerated, here too the laws of physics do not change for it. That is, Aristotle's approach is perfectly compatible with both Newtonian and Einsteinian-style physics. And certainly compatible with a physics that would claim that the tearing, understood as the first derivative of acceleration, that it undergoes is always constant.
    Aristotle's argument thus concerns physical reflection on symmetries as such (and which are, in fact, the laws of physics) and retains, certainly, a certain relevance.

    ReplyDelete
  33. In the realm of philosophy, it is crucial to examine the notion of contextuality in quantum mechanics and its corresponding Kochen-Specker Theorem, which carries the same weight as Bell's Inequalities. These concepts challenge traditional notions of determinism and can be more effectively understood within the framework of algebraic topology, as opposed to deterministic philosophical frameworks where they may appear as paradoxical. In reality, they serve to disprove deterministic approaches and force a reevaluation of our understanding of reality. which is totally compatible with A-T.

    ReplyDelete
    Replies
    1. Gaëtan Cantale-Miège,

      I have question about entanglement.

      If 2 entangled particles go in different directions and obey conservation of momentum then one will have up spin and the other will have down spin. But we won't know either's spin until we measure. Do I have that right?

      If they were not entangled, then we won't know either's spin even after we measure both.

      Why should we be surprised that entangled particles exhibit correlated behavior and that non-entangled particles exhibit non-correlated behavior?

      Delete
  34. (1) It is not the case that the two particles are in the correct bottom-up configuration from the start, and that our ignorance about this configuration is solely due to the fact that we have not yet performed the necessary measurements (this is the perspective of Newtonian deterministic physics). (2) In quantum mechanics, once produced, the two particles do not have an objectively defined "actual" configuration, but rather possess the objective "potentiality" to express one of the two possible configurations when measured. An apparent paradox arises only if we try to consider the two particles as being independent from each other after their production, because, in this case, once we measure the state of one of them, due to Einsteinian Relativity, there is no way that the result obtained by the measurement apparatus becomes instantly known by the other one which is now at a certain distance so that also this one can change its potential quantic state into a value which would be coherent with the one expressed by the particle measured first. (3) The apparent paradox comes from wanting to apply Newtonian categories, like determinism, to a Reality which, like in Aristotelian physics, does not need them in order to be fully operational. E.g., wanting to conceive the production of these particles as being the one of two actual particles is a gratuitous claim which comes from nowhere but is only supported by a too narrow Newtonian paradigm. (4) Instead of that, if you accept what is called non-locality and contextuality (exactly like we do accept it for the notion of potentiality in Aristotelian physics) then there is no paradox at all and entanglement becomes something ordinary we all experience everyday (I gave somewhere else the example of measuring the "breakability", which is a potentiality, of a bridge, "breakability" which is non-local and very much contextual and when that measurement happens, by breaking or not said bridge, this involves immediately the whole bridge independently from its length and the distance between its two opposite entrances)

    ReplyDelete
  35. Gaëtan Cantale-Miège,

    I understand QM doesn't contradict Aristotle and I understand the QM description of what happens to entangled particles before measurement.

    It seems to me from the description I've given above, that QM and classical agree that momentum is conserved among the 2 particles and that once one is measured we know the value of the other. There doesn't have to be any information transmitted between detectors in either case just the knowledge that the 2 particles were entangled to begin with and the ability to use the process of elimination.

    Why would a classicist think that information would need to be transmitted faster than the speed of light? The instant he measures the spin of one, he knows the spin of the other without any time lapse.

    ReplyDelete
    Replies
    1. Thank you for insisting and my apologies for not having answered your question earlier. There is a concept in quantum mechanics, as well as in our everyday world, that there are observables which are commutative and others which are not. When we observe physical states that are actual, our observables are commutative. However, when we measure potential properties of a physical state, we need to take two steps: (1) put the physical state in motion so that what is potential becomes actual, and (2) measure that actuality. Some of these motions from potentiality to actuality are reversible, but some others are not (for example, if we want to measure the breakability of a chalk, we need to break it, but then we no longer have a breakable chalk, but a broken one and no way to go back to the initial state). Similarly, there are measurable variables in quantum mechanics that measure actual or reversible potential properties, which are called commutative, and others that are irreversible and typically non-commutative. In quantum physics, spins or polarizations do not commute, nor do momentum and space position. For example, if we want to measure with extreme precision the momentum of our "two" famous particles, measuring one and then trying to measure with precision the position of the second one, this would not be possible, and this independently of the distance between the two particles at the moment of the measurement. The proof of the validity of this prediction of quantum mechanics was done by looking at the statistics of three non-commutative spin directions showing Bell's inequalities violation, which earned Aspect the 2022 Nobel Prize

      Delete
  36. Gaëtan Cantale-Miège,

    I understand that the Heisenberg Uncertainty Principle rules out precise measurements of both momentum and position at the same time. But this does not seem to be relevant to an observer measuring only the spin of one of a pair of entangled particles and by so doing knowing the spin of the other.

    The classicist could claim that since we know that the particles can only have 1 of 2 spin states and since they are entangled, the pair will have opposite spin states (due to conservation of momentum) during their trajectories to the detectors but we won't know which is up and which is down till at least one is detected and then the state of the other is known. The classicist makes the same accurate conclusion from the measurement results from one detector as the QM theorist. Neither requires superluminal information transfer so there is no paradox in either case.

    Again. Where is the paradox supposed to be here?

    The only disagreement is about what is really happening until the first measurement. But really finding out would require a measurement before the detector and that makes the disagreement unresolvable in principle.

    Maybe I have a misunderstanding of the entire entanglement story. Let me lay out my understanding point by point and talk about some issues I see.
    1) Entangled particle pairs differ from non-entangled particle pairs in that when measuring non-entangled particles there is no correlation between their 2 possible spin states while there is a correlation between the 2 possible spin states of entangled particle pairs. The correlation of entangled particle pairs is complementary in that if one is measured as up, the other is always measured as down (as in pi-meson decay)

    2) In the case of pi-meson decay, each particle must have complementary spin to preserve conservation of momentum.

    3) Experimentally we always observe that entangled particles behave this way.

    It seems to me that if some new experiments come up with different results, we have to conclude at least one of several things.

    A. Something is wrong with the experiment:
    The particles weren't entangled in the first place.
    The measurement devices were not set properly or were incapable of measuring accurately.
    Something unknown interfered with the experiment.
    There was an error in math when interpreting the results.

    B: Our theory is wrong
    We don't understand how entanglement works and so cannot reliably entangle pairs.
    We don't understand how our measurement devices work.
    Conservation of momentum is wrong.

    I think it's a bigger story if QM does away with conservation of momentum.

    ReplyDelete
    Replies
    1. Allow me to try again, as evidently, I have not been clear enough or too fast in my considerations. The main point of the Bell inequality experiments is about non-commuting variables. You can measure the vertical spin of an electron, and you can measure the horizontal spin, but you cannot measure both at the same time (just as you cannot measure a particle's momentum and position simultaneously). The correlation is not about saying that if particle 1's spin is "up" in the vertical direction, then particle 2's spin must be "down" in the same direction. The correlation is about saying that once the spin of particle 1 has been measured in the vertical direction, it becomes impossible to measure the horizontal spin of particle 2, as if the two particles were one, even though they are spatially separated. As you know, the probability for the first particle to be "up" could be, for example, 50%, and choosing to measure it randomly in any direction should satisfy Bell's inequalities if the two particles were independent of each other. If Bell's inequalities are falsified, it would mean that particle 1 has "informed" particle 2 instantaneously that it has been measured vertically, so that the latter cannot be measured horizontally. Experiments conducted by Nobel prize winners in 2022, such as A. Aspect, have shown that this is exactly what happens. In other words, the two particles are actually the expression of the same quantum state and the spin property of that quantum state is non-local, and it is incorrect to assume that these two particles can be considered as two independent (quantum) states that need to transfer information to each other.

      Delete